You are on page 1of 85

Abdelkader BENHARI

Exercises for Stochastic Calculus

Probability measure, Review of probability theory, Markov chains, Recurrence transition matrices,
Stationary distributions, Hitting times, Poisson processes, Renewal theory, Branching processes,
Branching and point processes, Martingales in discrete time, Brownian motion,
Martingales in continuous time

Series 1: Probability mesure

1.XERCISeX
Show that if
denition of

A and B
-algebra,

belongs to the

-algebra F

see Denition 1.3).

countable intersections, i.e. if

Ai F

for

B\A = B Ac = (B c A)c
(B A) F .
2) Take Ai F for i = 1, 2, . . . .
c c

(
i=1 A ) = i=1 A F

Proof. 1)

B\A F (for
F is closed under

i=1 Ai F .

then also

Also show that

i = 1, 2, . . .

, then

Bc F, Bc A F

and since

so

Since

i=1 A F

it follows that

2.
Though a fair die once. Assume that we only can observe if the number obtained
is small,

A = {1, 2, 3}

B = {1, 3, 5}. Describe the


-algebra F generated

and if the number is odd,

resulting probability space; in particular, describe the


by

and

in terms of a suitable partition (for denition of a partition, see

Denition 1.9) of the sample space.


Proof. Looking at the Venn diagram

insert diagram,

are at most four partitions of the space,

{5}

(A B)c = {4, 6} of which


4
combined in 2 = 16 dierent

and

can be

we conclude that there

A B = {1, 3}, A B c = {2}, Ac B =

none is an empty sets.


ways to generate the

These partitions

-algebra F

dened

below.

F ={, , A, B, Ac , B c , A B, A B c , Ac B, Ac B c ,
A B, A B c , Ac B, Ac B c , (A B c ) (Ac B), (A B) (Ac B c )}
={, , {1, 2, 3}, {1, 3, 5}, {4, 5, 6}, {2, 4, 6}, {1, 2, 3, 5}, {1, 2, 3, 4, 6},
{1, 3, 4, 5, 6}, {2, 4, 5, 6}, {1, 3}, {2}, {5}, {4, 6}, {2, 5}, {1, 3, 4, 6}}.
The probability measure on each of the sets in

may be deduced by using

the additivity of the probability measure and the probability measure of each

P (A B) = 2/6, P (A B c ) = 1/6, P (Ac B) = 1/6


P ((A B) ) = 2/6.
of the partitions,

and

3.
Given a probability space
Dene

(, F, P)

1. Show that

is

F -measurable

2. Find a special case where

and functions

X : R , Y : R.

Z = max{X, Y }.

if both

is

and

are

F -measurable.

measurable even though neither

is.

Proof. 1)

{ : Z() x} = { : max{X(), Y ()} x}


= { : X() x} { : X() x} F,
|
{z
} |
{z
}
F

nor

where the last conclusion is based on the result of proposition 3.2.

-algebra F was generated by the


A = {HT, T H}, i.e. F = {, , A, Ac } = {, {HH, T T, HT, T H}, {HT, T H}, {HH, T T }}.

2) Suppose you threw two coins and our


single set
Let

X=

1
0

if
if

{HT }
Y =
{HH, T T, T H}

1
0

{T H}
.
{HH, T T, HT }

if
if

Then

Z=
X

hence

and

are not

1
0

{HT, T H}
{HH, T T },

if
if

F -measurable

while

is.

4.

n = 4 times.
X : {1, 1}.

Toss a fair soin


functions

describe the sample space

1. describe the probability space

(, F1 , P1 )

come to be a legitimate event.


with

E[X] = 0

we want to consider

thar arises if we want each out-

How many

F1 -measurable

functions

are there?

2. Now describe the probability space

(, F2 , P2 )

that arises if we want that

only combinations of sets of the type

Ai = { :

Number of heads

should be events. How many

= i},

F2 -measurable

i = {0, 1, 2, 3, 4}

functions

with

E[X] = 0

are there?
3. Solve 2) above for some other

n.

Proof. 1) Each coin that is tossed has to possible outcomes, and since there

24 = 16 possible outcomes, or partitions.


So the sample space is = {HHHH, HHHT, HHT H, etc.} with 16 members
that describes all the information from that four tosses. F1 is the power set
of consisting of all combinations of sets of (don't forget that allways is
included in a -algebra ), hence F1 = (). P1 is deduced by rst observing that
each has P() = 1/16 and then using the additivity och the probability
are four coins to be tossed, there are

measure.
For any function
there is a set

A F1

X : {1, 1}

that is measurable with respect to

F1

such that

X() =

n 1
1

if
if

A
Ac .

E[X] = P1 (A) P1 (Ac ) so if E[X] = 0 we must have P1 (A) = P1 (Ac ),


since there are 16 partitions each with probability measure 1/16, there are

Hence

and
16
8 ways of combining the partitions so that there are equally many of them in
16
A and Ac , hence there are 8 possible nctions X : {1, 1} such that

E[X] = 0.

2) The sets

A0 = {T T T T }
A1 = {T T T H, T T HT, T HT T, HT T T }
A2 = {T T HH, T HHT, HHT T, HT T H, T HT H, HT HT }
A3 = {HHHT, HHT H, HT HH, T HHH}
A4 = {HHHH},
. The -algebra F2 is the -algebra generated by this
Pi = P(Ai ), then P0 = 1/16, P1 = 4/16, P2 = 6/16, P3 = 4/16
and P4 = 1/16. For any function X : {1, 1} that is measurable with
respect to F2 there is a a set A F2 such that
n 1
if
A
X() =
1 if Ac .
denes a partition of

partition. Dene

E[X] = P2 (A) P2 (Ac ) so if E[X] = 0 we must have P1 (A) = P1 (Ac ).


We may write E[X] = k0 P0 + k1 P1 + k2 P2 + k3 P3 + k4 P4 = k0 /160 + 4k1 /16 +
6k2 /16 + 4k3 /16 + k4 /16 which is zero either if ki = (1)i or if ki = (1)i .
Hence, there are two possible F2 -measurable functions X : {1, 1} such
that E[X] = 0.
Hence

3) Using the same notation as in 2), we conclude that, for any given in-

with -algebra Fn generated


by this partition, and with Pn (Ai ) =
i (1/2)i for i = 0, 1, . . . , n. And
n
n
Pn
Pn
E[X]
= i=0 ki i (1/2)i for ki = 1, where it should be noted that i=0 i

(1/2)i = 1. For any function X : {1, 1} that is measurable with respect
to Fn there is a a set A Fn such that
n 1
if
A
X() =
1 if Ac .
teger

n > 0, A0 , A1 , . . . , An

Hence

is a partition of

E[X] = Pn (A) Pn (Ac )

so if

E[X] = 0

we must have

Pn (A) = Pn (Ac ) =

1/2.
For even

n,

we have


i (1/2)i = 1/2

i=odd


i (1/2)i = 1/2

i=even
so

E[X] = 0

if

ki = (1)i

or

ki = (1)i .
(n1)/2

For odd

n,

we have


i (1/2)i = 1/2

i=0
n
X


i (1/2)i = 1/2

i=(n1)/2+1

i = 0, 1, . . . , (n 1)/2 and there are 2(n1)/2 ways


(n1)/2
of combining these. Hence, there are 2
possible Fn -measurable functions
X : {1, 1} such that E[X] = 0.
so

E[X] = 0 if kni = ki

for

5.
Consider the probability space
algebra on

and

(, F, P)

= [0, 1], F is the Borel (, F). Show that


Y () = 2| 1/2| have the same

where

is the uniform probability measure on

X() =
P(X = Y ) = 0.

the two random variables


distribution, but that

and

Proof. (For denition of distribution function see denition 3.4) Since

is

uniform, the distribution function is

P(X x) = 0
P(X x) = x
P(X x) = 1
Y

so for

we get, with

for
for
for

x (, 0]
x [0, 1]
x [1, )

x [0, 1],

P(Y x) = P(2|X 1/2| x) = P(|2X 1| x) = {by

symmetry}

= 2P(0 2X 1 x) = 2P(1/2 X (x + 1)/2)


= 2((x + 1)/2 1/2) = x.
Hence

P(X x) = P(Y x) = x

for

x [0, 1].

But since

P(X = Y ) = P(X = 2|X 1/2|)


= P({ : X = 2X 1/2} { : X = 2X + 1/2})
= P({ : X = 1} { : X = 3}) = P() = 0,
we conclude that

P (X = Y ) = 0.

6.

-algebra containing a set A is that intersection of all


A. Also, show by counter example that the union of two
necessarily a -algebra .

Show that the smallest

-algebras
-algebras

containing
is not

GA be the set of all -algebras containing A, we want to show that


GGA G = { : G for every G GA } = F is a -algebra ; if it is,
then it is the smallest -algebra since otherwise, there would be a -algebra
containing A that was smaller than F , this would be a contradiction since then
this -algebra would be included in G and therefore F would not be minimal.
To check that F is a -algebra , we simply use Denition 1.3.
Proof. Let

1.

so

G G GA

hence

F.
c G G GA

2. If

G G GA

3. If

1 , 2 , . . . G G GA

is a

-algebra

then

then

hence

c F .

i=1 i G G GA

hence

1 , 2 , . . . F .

-algebras is not necessarily a -algebra ,


F1 = {, , A, Ac } and F2 = {, , B, B c } where A B . Then F1 F2 =
{ : F1 or F2 } = {, , A, Ac , B, B c } is not a -algebra since, e.g.
Bc A
/ F1 F2 .
To show that the union of two

take

7.
given a probability space
algebra of

F,

(, F, P) and a random
= E[X|A].
X

variable

1. Show that

]=0
E[(X X)Y

if

is an

A-measurable

A-measurable random variable Y


Y = E[X|A].

2. Show that the


is

X.

Let

be a sub- -

and consider

random variable.

that minimize

E[(X Y )2 ]

Proof. 1) Using equality 4 followed by equality 2 in Proposition 4.8 we get

] = E[E[(X X)Y
|A]] = E[E[(X X)|A]Y

E[(X X)Y
]
X)Y
] = 0.
= E[(X
2) Using equality 3 followed by equality 2 in Proposition 4.8 we get

E[(X Y )2 ] = E[X 2 ] 2E[XY ] + E[Y 2 ]


= E[X 2 ] 2E[E[XY |A]] + E[Y 2 ]
= E[X 2 ] 2E[Y E[X|A]] + E[Y 2 ]
= E[X 2 ] E[E[X|A]2 ] + E[E[X|A]2 ] 2E[Y E[X|A]] + E[Y 2 ]
= E[X 2 ] E[E[X|A]2 ] + E[(Y E[X|A])2 ],
|
{z
}
0

E[(X Y )2 ]

hence

is minimized when

Y = E[X|A].

8.
Let

space

and Y be two integrable random variables dened on the same probability


(, F, P). Let A be a sub- -algebra such that X is A-measurable.

1. Show that

E[Y |A] = X

implies that

2. Show by counter example that


that

E[Y |X] = X .

E[Y |X] = X

does not necessarily imply

E[Y |A] = X .

Proof. (For denition of conditional expectation see Denition 4.6, for proper-

ties of the conditional expectation, see Proposition 4.8.)


1) Since

is

A-measurable, (X) A hence, using equality 4 in Proposition

4.8, we get

E[Y |X] = E[Y | (X)] = E[E[Y |A] |X] = E[X|X] = X.


| {z }
| {z }
A

=X

X and Z be independent and integrable random variables and assume


E[Z] = 0 and dene Y = X + Z . Let A = (X, Z), then

2) Let
that

E[Y |(X)] = E[X + Z|(X)] = X + 0 = X,


while

E[Y |A] = E[X + Z|(X, Z)] = X + Z 6= X.

Series 2 : review of probability theory

Exercise 1
For each given p let X have a binomial distribution with parameters p and N.
Suppose N is itself binomially distributed with parameters q and M, M > N.
(a) Show analytically that X has a binomial distribution with parameters pq and M.
(b) Give a probabilistic argument for this result.
Exercise 2
Using the central limit theorem for suitable Poisson random variables, prove
that
n

lim e

n
X
nk
k=0

1
= .
k! 2

Exercise 3
Let X and Y be independent, identically distributed, positive random variables
with continuous density function f (x) satisfying f (x) > 0 for x > 0. Assume, further, that U = X Y
and V = min(X, Y) are independent random variables. Prove that
( x
e
for x > 0;
f (x) =
0
elsewhere,
for some > 0.
Hint: Show first that the joint density function of U and V is
fU,V (u, v) = f (v) f (v + |u|).
Next, equate this with the product of the marginal densities for U, V.
Exercise 4
Let X be a nonnegative integer-valued random variable with probability
P
n
generating function f (s) =
n=0 an s . After observing X, then conduct X binomial trials with probability
p of success. Let Y denote the resulting number of successes.
(a) Determine the probability generating function of Y.
(b) Determine the probability generating function of X given that Y = X.
(c) Suppose that for every p (0 < p < 1) the probability generating functions of (a) and (b) coincide.
Prove that the distribution of X is Poisson, i.e., f (s) = e(s1) for some > 0.

Series 2: review of probability theory

Solutions

Exercise 1
(a) Since the distribution of N is binomial(M, q), we have
fN (s) = E[sN ] = (sq + 1 q)M .
Since the conditional distribution of X given N = n is Binomial(n, p), we have
fX|N=n (s) = E[sX |N = n] = (sp + 1 p)n .
We can compute the generating function of X, say fX (s), by conditioning on N. We obtain
X

fX (s) = E[s ] =

M
X
n=0

E[sX | N = n] P[N = n]

M
X
=
(sp + 1 p)n P[N = n] = fN (sp + 1 p)
n=0



(sp + 1 p)q + 1 q M = s(pq) + 1 pq M .

Here we recognize the generating function of the binomial(M, pq) distribution.


Heres another approach. For k {0, 1, 2, ..., M}, we get
P[X = k] =

M
X

n=0
M
X
n=k

P[X = k| N = n] P[N = n]
!
!
n k
nk M n
p (1 p)
q (1 q)Mn
k
n
M

=
=

X
(M k)!
M!
(pq)k
((1 p)q)nk (1 q)Mk
k!(M k)!
(n

k)!(M

n)!
n=k
!
!
Mk
X
M
Mk
k
(pq)
((1 p)q) j (1 q)Mk j
k
j
j=0
!
!
M
M
k
Mk
(pq) ((1 p)q + (1 q))
=
(pq)k (1 pq) Mk .
k
k

This is the binomial(M, pq) distribution.


(b) Imagine M kids. Each one of them will toss a silver coin and a gold coin. With a silver coin the
probability of heads is q. With a gold coin, it is p. Let N be the number of kids who will get heads
with the silver coin and let X be the number of kids who will get heads on both tosses. Clearly the
distribution of N is binomial(M, q), the conditional distribution of X given N = n is binomial(n, p) and
the distribution of X is binomial(M, pq).

Exercise 2
Let X , X , X , ... be independent random variables, each with distribution Poisson(1), and let S n =
Pn 1 2 3
k=1 Xk . Since the mean and the variance of the Poisson(1) distribution are both equal to 1, the central limit theorem gives us
"
# Z x
2
Sn n
1
lim P
x =
eu /2 du
n
n

2
for every x R. With x = 0, this gives us
"

#
Sn n
1
lim P
0 = .
n
2
n
This last equation can be written as
1
lim P [S n n] = .
n
2
In view of the lemma below, the distribution of S n is Poisson(n). Therefore, the last equation can be
written as
n
X
nk 1
n
= .
lim e
n
k! 2
k=0
Lemma. If U Poisson() and V Poisson() and if U and V are independent, then U + V
Poisson( + ).
The elementary proof is left as an exercise.
Exercise 3
Let fX,Y (x, y) denote the joint density of X and Y. Let fU,V (u, v) denote the joint density of U and V. For
the moment we do not make any independence assumptions.
Fix u > 0 and v > 0. The following computation will be easy to follow if you look at the first quadrant
of R2 and visualize the set of points (x, y) for which min(x, y) v and x y u.
fU,V (u, v) =
=
=

2
P[(U u) (V v)]
uv
2
P[(X Y u) (min(X, Y) v)]
uv
2
(P[(X, Y) A] + P[(X, Y) B])
uv

where A = {(x, y) R2 : v < y < and 0 < x < v} and B = {(x, y) R2 : 0 < y v and 0 < x u + y}.
Since A depends only on v, we have
2
P[(X, Y) A] = 0.
uv
Thus we have
fU,V (u, v) =
=
=
=

2
P[(X, Y) B]
uv
Z v Z u+y
2
fX,Y (x, y) dx dy
uv 0 0
Z u+v

fX,Y (x, v) dx
u 0
fX,Y (u + v, v).

A similar calculation gives us fU,V (u, v) = fX,Y (v u, v) for the case where u > 0 and v < 0. (You should
do the computation; be careful with the domain of integration in the (x, y) plane). In summary, we have
shown that
(
fX,Y (v + |u|, v) if u R and v > 0,
(1)
fU,V (u, v) =
0
otherwise.
If we assume that X and Y are independent and identically distributed with density f , then equation (1)
can be written as
(
f (v + |u|) f (v) if u R and v > 0,
fU,V (u, v) =
0
otherwise.
If we assume furthermore that U and V are independent, i.e., fU,V (u, v) = fU (u) fV (v) for all v > 0 and
u R, then the above result gives us
f (v) f (v + |u|) = fU (u) fV (v)

v > 0, < u < .

In particular we have
(2)

f (u + v) = g(u) h(v)

u > 0, v > 0

with g(u) = fU (u) and h(v) = fV (v)/ f (v). From (2) we get, for all x > 0 and y > 0,
R
R
R
f
(x
+
v)dv
h(v)dv
h(v)dv
g(x)
P[X > x + y]
y
y
y
R
= R
=
= R
P[X > x]
f (x + v)dv g(x)
h(v)dv
h(v)dv
0

Thus the ratio P[X > x + y]/P[X > x] does not depend on x. Taking the limit as x 0, we see that this
ratio is equal to P[X > y]. Thus we have
P[X > x + y] = P[X > x] P[X > y]

(3)

pour tout x > 0, y > 0.

The lemma below allows us to conclude that X exponential().


Lemma. Let X be a non negative random variable.
(a) If X exponential() for some > 0, then equation (3) is true.
(b) If equation (3) is true, then X exponential() (for some > 0).
The proof is left to the reader.
Exercise 4
(a) The distribution of Y given X = n is Binomial(n, p). Thus,
E(sY |X = n) = (sp + 1 p)n .
Using this, we have
fY (s) = E(sY ) =

X
n=0

E(sY |X = n) P(X = n) =

(b) We have
P(X = Y) =

X
n=0

X
(sp + 1 p)n P(X = n) = f (sp + 1 p).
n=0

P(X = n, Y = n) =

pn P(X = n) = f (p)

n=0

and then
P(X = n | X = Y) =

P(X = n, X = Y) P(X = n)P(Y = n | X = n) pn P(X = n)


=
=
.
P(X = Y)
f (p)
f (p)

To conclude,
fX|X=Y (s) =

X
n=0

sn P(X = n | X = Y) =

1 X n n
f (sp)
.
s p P(X = n) =
f (p) n=0
f (p)

(c) From parts (a) and (b) we have


f (1 p + ps) =

f (ps)
f (p)

0 < p < 1 and 1 < s < 1.

If we fix p and we take derivative w.r.t. s, we obtain


f (1 p + ps) p =

f (ps) p
f (p)

0 < p < 1 and 1 < s < 1.

Now we divide by p on both sides and we evaluate at s = 1. We obtain


f (1) =

f (p)
f (p)

0<p<1

E[X] =

f (p)
f (p)

0< p<1

i.e.
(4)

When we solve equation (4), subject to the boundary condition f (1) = 1, we get f (s) = e(s1) (where
= E[X]). We conclude that the distribution of X is Poisson.

10

Series 3: Markov chains

Exercise 1
Determine the classes and the periodicity of the various states for
a Markov chain with transition probability matrix



0 0 1 0
0 1 0 0
1 0 0 0
0 0 0 1
,
(a) 1 1
(b)
2 2 0 0
0 1 0 0
1 1 1 0
1 0 2 0
3
3
3
3
3

Exercise 2
Consider repeated independent trials of two outcomes S (sucess)
or F (failure) with probabilities p and q, respectively. Determine the mean of the number of trials required
for the first occurence of the event SF (i.e., sucess followed by failure). Do the same for the event SSF
and SFS.
Exercise 3
Let a Markov chain contain r states. Prove the following:
(a) If a state k can be reached from j, then it can be reached in r 1 steps or less.
(b) If j is a recurrent state, there exists (0 < < 1) such that for n > r the probability that the first
return to state j occurs after n transitions is 6 n .
Exercise 4
In this exercise, consider a random walk on the integers such that Pi,i+1 = p,
Pi,i1 = q for all integer i (0 < p < 1, p + q = 1).
(a) Determine Pn0,0 .
(b) For any real a and natural n, define
!
a
a(a 1) (a n + 1)
.
=
n!
n
Prove that

!
!
2n
n 1/2 2n
= (1)
2 .
n
n

P
n
(c) Let P be the generating function associated to the sequence un = Pn0,0 , i.e., P(x) =
n=0 un x . Show
that P(x) = (1 4pqx2 )1/2 .
(d) Prove that the generating function of the recurrence time from state 0 to state 0 is F(x) = 1
p
1 4pqx2 .
(e) What is the probability of eventual return to the origin?

11

Exercise 5
A player has k coins. At each round independently, he gains one coin with probability p, or loses one
coin with probability 1 p = q. He stops to play if he reaches 0 (he loses) or N (he wins). What is the
probability to win ?
Exercise 6
Suppose X1 , X2 , . . . are independent with P(Xk = +1) = p, P(Xk = 1) = q = 1 p, where p > q
With S 0 = 0, set S n = X1 + + Xn , Mn = max{S k : 0 6 k 6 n} and Yn = Mn S n .
If T (a) = min{n : S n = a}, show
 y a

if p = q = 1/2;

1+y

"  y+1 #a
p
p
max Yk < y =
P 06k6T

q q
(a)

if p , q
 p y+1

1
q

Hint: The bivariate process (Mn , Yn ) is a random walk on the positive lattice. What is the probability that
this random walk, started at (0, 0), leaves the rectangle in the picture from the top?

12

Solutions 3

Solutions

Exercise 1
(a) We have 1 3, 3 2 and 2 1, so 1, 2 and 3 are in the same class. Also, for states i = 1, 2, 3
we have P(i, 4) = 0, so class {1, 2, 3} is recurrent. Since P(4, 4) = 0, 4 itself forms a transient class. The
classes are thus {1, 2, 3}, {4}. Periodicity is only defined for recurrent states, so it suffices to determine the
period of 1, 2 and 3. Since they are in the same class, they have the same period, so it suffices to determine
the period of 1. We have P2 (1, 1) > P(1, 3)P(3, 1) > 0 and P3 (1, 1) = P(1, 3)P(3, 2)P(2, 1) > 0, so the
period of 1 must divide 2 and 3, hence it is equal to 1.
(b) Since 1 2, 2 4, 4 3, 4 1 and 3 2, there is a single recurrent class {1, 2, 3, 4}. Let us
determine the period of state 4. We have P(4, 4)P2 (4, 4) = 0. Also, P3 (4, 4) = 1, so P3n (4, 4) = 1 for all
n. Fix N N, N > 3 and N not divisible by 3; we can then write N = 3n + k for some n N and k = 1
or 2. Then,
4
X
N
3n+k
P (4, 4) = P
(4, 4) =
P3n (4, i)Pk (i, 4) = P3n (4, 4)Pk (4, 4) = 0.
i=1

We thus have
3.

PN (4, 4)

> 0 if and only if N is a multiple of 3, so the period of 4 (and every other state) is

Exercise 2
Let 1 , 2 , . . . be a sequence of letters chosen independently at random so that for each i, P(i = S ) = p =
1 q = 1 P(i = F).
Time until SF. Let X0 = , X1 = 1 and Xn = n1 n for n > 2, where in each case we are simply
concatenating the symbols. (Xn ) is then a Markov chain on the state space {, S , S , S S , S F, FS , FF}.
Define, for n > 0,
an = P (min{k : Xk = S F} = n), bn = P (Xn = S F), cn = PS F (Xn = S F);

X
X
X
n
n
F,S F (s) =
an s , P,S F (s) =
bn s , PS F,S F (s) =
cn sn ,
n=0

n=0

n=0

where s C with |s| < 1. We have


b0 = b1 = 0, bn = pq

n > 2;

c0 = 1, c1 = 0, cn = pq
so
P,S F (s) =

X
n=2

pqs2
,
pqs =
1s
n

n > 2,

PS F,S F (s) = 1 +

pqsn = 1 +

n=2

Using the formula F x,y (s)Py,y (s) = P x,y (s) (which applies when x , y, we have
F,S F (s) =

pqs2
1s
2
1 + pqs
1s

pqs2
.
1 s + pqs2

Then,

F,S
F (s) =

13

2pqs(1 s + pqs2 ) pqs2 (1 + 2pqs)


(1 s + pqs2 )2

pqs2
.
1s


and F,S
F (1) = 1/pq. We are now done:

E(min{n : Xn = S F}) = F,S


F (1) = 1/pq.

Time until SSF. Now put Y0 = , Y1 = 1 , Y2 = 1 2 and Yn = n2 n1 n for n > 3. Define


an = P (min{k : Xk = S S F} = n), bn = P (Xn = S S F), cn = PS S F (Xn = S S F);

X
X
X
n
n
F,S S F (s) =
an s , P,S S F (s) =
bn s , PS S F,S S F (s) =
cn sn .
n=0

n=0

We have

n=0

b0 = b1 = b2 = 0, bn = p2 q n > 3;
c0 = 1, c1 = c2 = 0, cn = p2 q

n > 3,

so that
P,S S F (s) =

X
n=3

p2 qs3
,
p qs =
1s
2

PS S F,S S F (s) = 1 +

p2 qs3 = 1 +

n=3

and
F,S S F (s) =
We thus have

p2 qs3
1s

p2 qs3
.
1 s + p2 qs3

3p2 qs2 (1 s + p2 qs3 ) p2 qs3 (1 + 3p2 qs2 )


(1 s + p2 qs3 )2

F,S
S F (s) =

and F,S
S F (1) = 1/p q = E(min{n : Xn = S S F}).
Time until SFS. Similarly to before, put

an = P (min{k : Xk = S FS } = n), bn = P (Xn = S FS ), cn = PS FS (Xn = S FS );

X
X
X
n
n
F,S FS (s) =
an s , P,S FS (s) =
bn s , PS FS ,S FS (s) =
cn sn .
n=0

We now have

n=0

n=0

b0 = b1 = b2 = 0, bn = p2 q n > 3;
c0 = 1, c1 = 0, c2 = pq, cn = p2 q n > 3,

so that
P,S FS (s) =

p2 qs3 =

n=3

p2 qs3
,
1s

and
F,S FS (s) =

PS FS ,S FS (s) = 1 + pqs2 +

p2 qs3 = 1 + pqs2 +

n=3

p2 qs3
1s

1 + pqs2 +

p2 qs3
1s

p2 qs3
1s

p2 qs3
.
1 s + pqs2 + (p2 q pq)s3

Finally,
3p2 qs2 (1 s + pqs2 + (p2 q pq)s3 ) p2 qs3 (1 + 2pqs + 3(p2 q pq)s2 ))
;
(1 s + pqs2 + (p2 q pq)s3 )2
3p2 q(pq + p2 q pq) p2 q(1 + 2pq + 3p2 q 3pq)
1
1

(1)
=
F,S
= 2 + ,
FS
2
2
(pq + p q pq)
p q p

F,S
FS (s) =

so E(min{n : Yn = S FS }) =

1
p2 q

+ 1p .

14

Exercise 3
(a) We assume that k , j. The fact that k can be reached from j means that there exists a sequence of
states x0 , x1 , . . . , xN such that x0 = j, xN = k and P(xi , xi+1 ) > 0 for all i. Define
= {i {1, . . . , N 1} : j1 {1, . . . , i}, j2 {i + 1, . . . , N} : x j1 = x j2 }
(the set of indices i such that xi is between two repeated entries in the sequence x1 , . . . , xN ). By enumerating {0, 1, . . . , N}\ = {a0 , . . . , aM } with a0 = 0 < a1 < . . . < aM = N, it is easy to see that P(xai , xai+1 ) > 0
for each i and that xa0 , xa1 , xa2 , . . . xaM has no repetitions. This implies that this sequence has at most r
elements, so the path xa0 xa1 . . . xaM1 xaM shows that k can be reached from j in at most
r 1 steps.
(b) Define j = min{n > 1 : Xn = j}. Assume that j k. Since j is recurrent, we must also have k j,
and by part (a), j can be reached from k in at most r 1 steps. This implies that
Pk ( j 6 r 1) > 0
or, equivalently, that
Pk ( j > r) < 1.
Since the state space is finite, we thus conclude that the number
n
o
= min Pk ( j > r) : k can be reached from j

is smaller than 1. Now, fix n N. We have n/rr 1 < n, so the event { j > n} is contained in
{ j > n/rr 1}. Using this, we have
P j ( j > n) 6 P j ( j > n/rr 1) = P j ( j < {X1 , . . . , Xn/rr1 })
X
=
P j ( j < {X1 , . . . , X(n/r1)r1 }, X(n/r1)r1 = k) Pk ( j > r)
k: jk

P j ( j < {X1 , . . . , X(n/r1)r1 }, X(n/r1)r1 = k)

k: jk

= P j ( j > (n/r 1)r 1).


Proceeding inductively, we get
P j ( j > n) 6 n/r ,

n>r

We can now choose (0, 1) such that, for any n > r,


n/r 6 n ,
completing the proof.
Exercise 4
(a) Let 1 , 2 , . . . denote a sequence of independent and identically distributed random variables with
P
P(i = 1) = q, P(i = 1) = p for each i. Putting X0 0 and Xn = ni=1 n , we see that Xn is a Markov
chain with transitions given by P. Since for any n > 1 we have Xn Xn+1 {1, 1}, Xn has the same
parity as n. As a consequence, Pn (0, 0) = 0 when n is odd. If n is even, then
n

!
X
X

n
P(1 = y1, . . . , n = yn ) = n/2 pn/2 qn/2 .
P(Xn = 0) = P i = 0 =
P
i=1
y ,...,y {1,1}n : y =0
1

3
2
1
n
2
1
n
2

(b)

!
!
1/2
1 1

=
n!
2
n
1
= (1)n
n!
n 1
= (1)
n!

15

(2n 1)

1 3 5 (2n 1)

1 2 3 4 2n
1 1 (2n)!
= (1)n n n
2 4 6 2n
n! 2 2 n!

Then,

!
!
2n (2n)!
1/2 2n
2n
2n 1 2
(1)
2 = (1) 2n
=
n
n! 2
n!
n
n

(c) By part (a),


P(x) =

u2n x2n =

n=0

X
2n
(pqx2 )n .
n
n=0

Using (b), we obtain


!

X
1/2
P(x) =
(4pqx2 )n
n
n=0
= (1 4pqx2 )1/2 .
(d) Using the formula F(x)P(x) = P(x) 1 and part (c), we immediately get F(x) = 1
P
k
(e) Writing F(x) =
k=0 fk x , we have
P(n : Xn = 0) =

P(min{n : Xn = 0} = k) =

k=0

fk = F(1) = 1

k=0

1 4pqx2 .

p
1 4pq.

Exercise 5
Let (Xn )nN be the Markov chain representing the fortune of the player, and let be the time when the
game ends, that is,
= inf{n 0 : Xn = 0 or Xn = N}.
Let uk be the probability that starting from k coins, the player wins, that is,
uk = Pk [X = N],
where Pk = P[ | X0 = k]. For 0 < k < N, we have
Pk [X = N] = Pk [X1 = k 1, X = N] + Pk [X1 = k + 1, X = N].
Note that this is equal to
Pk [X1 = k 1] [X = N | X1 = k 1] + Pk [X1 = k + 1] Pk [X = N | X1 = k + 1]
= q Pk1 [X = N] + p Pk+1 [X = N],
where we used the Markov property in the last step. We have proved that
uk = quk1 + puk+1 .
Moreover, it is clear that u0 = 0 and uN = 1. We solve this recurrence equation as in Karlin-Taylor, (3.6)
in chapter 3. For p = 1/2, we obtain
k
uk = ,
N
and otherwise,
1 (q/p)k
uk =
.
1 (q/p)N
Exercise 6
From the fact that (S n ) is a Markov chain, it follows that ((Yn , Mn )) is also a Markov chain. To justify
this properly, fix a sequence (y0 , m0 ) = (0, 0), (y1 , m1 ), . . . , (yn , mn ); noticing the equality
{(Y0 , M0 ) = (0, 0), (Y1 , M1 ) = (y1 , m1 ), . . . , (Yn , Mn ) = (yn , mn )} = {S 0 = 0, S 1 = m1 y1 , . . . , S n = mn yn },

16

we have
P((Yn+1 , Mn+1 ) = (y, m) | (Y0 , M0 ) = (0, 0), . . . , (Yn , Mn ) = (yn , mn ))
!

max06i6n+1 S i = m,

=P
S 0 = 0, S 1 = m1 y1 , . . . , S n = mn yn
(max06i6n+1 S i ) S n+1 = y

p if y = yn = 0, m = mn + 1;

or if y > 0, y = yn 1, m = mn ;

q if y = yn + 1, m = mn ;

0 in any other case.

In words, (Yn , Mn ) takes values in the positive quadrant of Z2 (see the picture in the statement of the
exercise) and moves according to the rules:
if y > 0, we jump to the right with probability q and to the left with probability p;
if y = 0, we jump to the right with probability q and up with probability p.
Notice that we never jump down and we can only jump up on the vertical axis.
For a > 0, we have T (a) = min{n : S n = a} = min{n : Mn = a}, so
(
)


max Yk < y = (Yn , Mn ) reaches (a, 0) before reaching {y} Z+ ,
06k6T (a)

so, using the Strong Markov Property,


P

! Y
a1
max Yk < y =
P

06k6T (a)

i=0

=P

!
(Yn , Mn ) reaches (0, i + 1)
(Y0 , M0 ) = (0, i)
before reaching (y, i)
!a
(Yn , Mn ) reaches (0, a)
(Y0 , M0 ) = (0, a 1)
before reaching (y, a 1)

The probability in the above expression is the probability that the chain in the picture reaches the upper
point (0, a) before reaching the rightmost point (y, a 1). From the gamblers ruin problem, we see that
this is equal to
y

if p = q = 1/2;

1+y

p  p y+1

q q

 p y+1 if p , q,
1

finishing the proof.

17

Series 4: recurrence, transition matrices

Exercise 1
Let P be the transition matrix of a Markov chain. If j is a transient state, prove
that for all i,
+
X
Pni, j < +.
n=1

Exercise 2
Consider a sequence of Bernoulli trials X1 , X2 , . . ., where Xn = 1 or 0. Assume
that there exists > 0 such that, for any n,
P [Xn = 1 | X1 , . . . , Xn1 ] .
Show that
(1) P[Xn = 1 for some n] = 1,
(2) P[Xn = 1 infinitely often] = 1.
Exercise 3
Players I, with fortune a, and player II, with fortune b, play
together (a > 10, b > 10). At each round, player I has probability p to win one unit from player II,
else he gives one unit to player II. What is the probability that player I will achieve a fortune a + b 3
before his fortune dwindles to 5?
Exercise 4
A car can be in one of three states: working (state 1), being repared (state 2), or definitely out of order
(state 3). If the car is working on some day, then the probability it will be working the day after is
0.995, and the probability it will need repair is 0.005. If it is being repaired, then the probability it will
be working the day after is 0.9, the probability it will still need repair is 0.05, the probability it will be
definitely out of order is 0.05.
(1) Write the transition matrix for this Markov chain.
(2) Let us define the reduced matrix
"
#
0.995 0.005
R=
.
0.9
0.05
Show that Q = (I R)1 is well defined, and that Qi j is the expected total time spent at state j starting
P
n
from state i (Hint: study the series +
n=0 R . To show that Q and this series are well defined, you may
study the spectrum of the transpose of R.)
(3) Starting from a car that is working, what is the expected time before it gets definitely out of order ?
(4) How can the result be extended to more general Markov chains ?
(5) Use this method to solve exercise 2 of the previous series.

18

Series 4: recurrence, transition matrices

Solutions

Exercise 1
Let us write (Xn ) for the Markov chain with transition matrix P, Pi for P[ | X0 = i], and
i = inf{n 1 : Xn = i}.
Let
Finj = Pi [ j = k],
P+ n n
P
n n
and let Fi j (s) be its generating function, Fi j (s) = +
n=0 Pi j s . One can
n=0 F i j s . Let also Pi j (s) =
show that
1
,
P j j (s) =
1 F j j (s)
see Karlin-Taylor, Eq. (5.8) of Chapter 2 for a proof. By definition, if the state j is transient, then
P j [ j < +] < 1.
This amounts to saying that
+
X

F njj < 1,

n=0

and thus
lim P j j (s) < +.
s1

We have thus proved that


+
X

Pnj j < +.

n=0

For i , j, we have
Pi j (s) = Fi j (s)P j j (s)
(see Karlin-Taylor, Eq. (5.10) of Chapter 2), and thus we also have
lim Pi j (s) < +,
s1

or equivalently,
+
X

Pnij < +.

n=0

Exercise 2
(1) Note that
P[X1 = = Xn = 0] = P[Xn = 0 | Xn1 = = X1 = 0] P[Xn1 = = X1 = 0]
(1 ) P[Xn1 = = X1 = 0].
By induction, we thus get
P[X1 = = Xn = 0] (1 )n ,

19

and as a consequence,
P[n {1, 2, . . .}, Xn = 0] = 0.
(2) If it is not true that Xn = 1 infinitely often, then there exists N such that for all n N, Xn = 0. We
have
+
X
P[N : n N, Xn = 0]
P[n N, Xn = 0],
N=0

and we proceed as in (1) to show that each summand is equal to 0.


Exercise 3
Let (Xn )n>0 be a Markov chain on Z such that, for each n, the transitions n n + 1 and n n 1 occur
with probability p and 1 p, respectively. We want to find
P(Xn reaches a + b 3 before reaching 5 | X0 = a),
which is the same as
P(Xn reaches a + b 8 before reaching 0 | X0 = a 5).
By Gamblers ruin, this is equal to

 q a5

1
p
 q a+b8
1
p
a5
a+b8

if p , q;
if p = q = 1/2.

Exercise 4
(1) The transition matrix is

0
0.995 0.005

0.05 0.05 .
P = 0.9

0
0
1

(2) Remark: for the purpose of this question, computing explicitely the eigenvalues of this 2 2 matrix
and checking that their modulus are strictly smaller than 1 would be fine. Here, we study the spectrum
using arguments that are easier to extend to more general situations.
Note that for any i, one has
X
Ri j 1,
j

Hence, for any x = (x1 , x2 )


(1)

C2,



X X
XX

Ri j xi
Ri j |xi | kxk1 .
kR xk1 =


T

As a consequence, the spectrum of RT (which is the same as that of R) is contained in {


Moreover, since
X
R2 j < 1,

C : || 1}.

the inequality in (1) is strict if x2 , 0.


Assume that there exists of modulus 1, and x 2 a non-zero vector such that RT x = x. Then it
is easy to see that x2 , 0, and thus kRT xk1 < kxk1 , a contradiction with the assumption that has modulus
one. We have proved that the eigenvalues of RT , which are also those of R, have modulus strictly smaller
than 1.

20

This clearly implies that Q and the series


+
X

Rn

n=0

are well defined. Now, observe that


(I R)

+
X

Rn =

n=0

+
X
n=0

Rn

+
X

Rn = I,

n=1

P
n
so in fact Q = +
n=0 R .
Finally, one has to see that Rnij is the probability that starting from i, the walk is at j at time n. By
definition, this probability is
X
Pi0 i1 Pin1 in ,
i=i0 ,i1 ,...,in1 ,in = j

where i1 , . . . , in1 take all possible values in {1, 2, 3}. But since state 3 is absorbing (that is, P3 j = 0 if
j , 3), the sum above is zero if one ik is equal to 3. So the sum is equal to
X
Ri0 i1 Rin1 in ,
i=i0 ,i1 ,...,in1 ,in = j

where i1 , . . . , in1 take all possible values in {1, 2}. This last expression is precisely Rnij .
(3) A computation shows that
"
#
3800 20
Q=
.
3600 20
Starting from a car that is working, the expected time before it goes out of order is the sum of expected
times spent in states 1 and 2, which is thus 3820 days (and on average, the total number of days during
which the car is working before it gets out of order is 3800 days).
(4) A careful examination of the proof of part (2) shows that the only requirement is that the out of
order state can be reached from any other state in a finite number of steps.
(5) For instance, to get the expected time before getting SF, we may consider the (3 3) reduced
transition matrix R given by
FF FS S S
FF q
p
0
.
FS
0
0
p
0
0
p
SS
For the inverse, we obtain
(I R)1

1/p 1 p/q

= 0 1 p/q .

0 0 1/q

It takes two tosses to get the chain started. But the moment of appearance of SF is not changed if
instead we start in the state FF. We get that the expected time before SF appears is
1
p 1 1
+1+ = + ,
p
q
p q
as expected.

21

Series 5: stationary distributions

Exercise 1
Consider a Markov chain with transition probabilities matrix

p0
p
P = m

p1

p1
p0

p2

p2
p1

p3

pm
pm1

p0

where 0 < pi < 1 for each i and p0 + p1 + + pm = 1. Determine lim Pnij , the stationary distribution.
n

Exercise 2
An airline reservation system has two computers only one of which
is in operation at any given time. A computer may break down any given day with probability p. There
is a single repair facility which takes 2 days to restore a computer to normal. The facilities are such that
only one computer at a time can be dealt with. Form a Markov chain by taking as states the pairs (x, y)
where x is the number of machines in operating conditions at the end of a day and y is 1 if a days labor
has been expended on a machine not yet repaired and 0 otherwise. Enumerating the states in the order
(2, 0), (1, 0), (1, 1), (0, 1), the transition matrix is

q p 0
0 0 q

q p 0

0 1 0

0
p
0
0

where p, q > 0 and p + q = 1. Find the stationary distribution in terms of p and q.


Exercise 3
Sociologists often assume that the social classes of successive
generations in a family can be regarded as a Markov chain. Thus, the occupation of a son is assumed
to depend only on his fathers occupation and not on his grandfathers. Suppose that such a model is
appropriate and that the transition probability matrix is given by

Fathers class

Lower
Middle
Upper

Lower
.40
.05
.05

Sons class
Middle Upper
.50
.10
.70
.25
.50
.45

For such a population, what fraction of people are middle class in the long run?
Exercise 4
Suppose that the weather on any day depends on the weather
conditions for the previous two days. To be exact, suppose that if it was sunny today and yesterday, then
it will be sunny tomorrow with probability .8; if it was sunny today but cloudy yesterday, then it will
be sunny tomorrow with probability .6; if it was cloudy today but sunny yesterday, then it will be sunny
tomorrow with probability .4; if it was cloudy for the last two days, then it will be sunny tomorrow with
probability .1.

22

Such a model can be transformed into a Markov chain provided we say that the state at any time is
determined by the weather conditions during both that day and the previous day. We say the process is
in
State (S,S) if it was sunny both today and yesterday;
State (S,C) if it was sunny yesterday but cloudy today;
State (C,S) if it was cloudy yesterday but sunny today;
State (C,C) if it was cloudy both today and yesterday.
Enumerating the states in the order (S,S), (S,C), (C,S), (C,C), the transition matrix is



.8 .2

.4 .6



.6 .4

.1 .9
Find the stationary distribution for the Markov chain.
Exercise 5
(Chapter 3, Problem 4) Consider a discrete time Markov chain with states 0, 1, . . . , N whose matrix has
elements

i ,
j = i 1,

i ,
j = i + 1, i, j = 0, 1, . . . , N;

Pi j =

1 i i ,
j = i;

0,
| j i| > 1.
Suppose that 0 = 0 = N = N = 0, and all other i s and i s are strictly positive, and that the initial
state of the process is k. Determine the absorption probabilities at 0 and N.

23

Series 5: stationary distributions

Solutions

Exercise 1
Since all entries in the transition matrix are strictly positive, the chain is irreducible and positive recurrent,
so there exists a unique invariant measure. By the symmetric structure of the matrix, we can guess that
1
1
, . . . , m+1
) is invariant, and indeed it is trivial to check that P = .
the uniform distribution = ( m+1
Exercise 2
Following the enumeration in the exercise, let (1 , 2 , 3 , 4 ) denote the stationary distribution. It must
satisfy:
(1)

1 + 2 + 3 + 4 = 1;

(2)

q1 + q3 = 1 ;

(3)

p1 + p3 + 4 = 2 ;

(4)

q2 = 3 ;

(5)

p2 = 4 .

Applying (4) in (2), we get


q1 + q2 2 = 1 = 1 =

q2
2 .
p

Using this, (4) and (5) in (1), we get


q2
p
.
2 + 2 + p2 + q2 = 1 = 2 =
p
1 + p2
We thus get 1 =

q2
,
1+p2

2 =

p
,
1+p2

3 =

qp
,
1+p2

4 =

p2
.
1+p2

Exercise 3
Since all the entries of the transition matrix are positive, the chain is irreducible and aperiodic, so there
exists a unique stationary distribution. Denoting this distribution by = (x, y, z), the quantity we are
looking for namely, the fraction of people that are middle class in the long run is equal to y. We know
that x + y + z = 1 and also that

.40 .50 .10

(x y z) .05 .70 .25 = (x y z);

.05 .50 .45


solving the corresponding system of linear equations, we get x =

1
13 ,

y=

5
8

and z =

31
104 .

Exercise 4
Let x, y, z, w denote the states of the chain in the order that they appear in the transition matrix. The
invariant measure is obtained by solving the system

.8 .2

.4
.6
= (x y z w);
(x y z w)

.6 .4

.1 .9
x + y + z + w = 1.

24

3
The unique solution is (x y z w) = ( 11
,

1
1
6
11 , 11 , 11 ).

Exercise 5
For k {0, . . . , N}, define f (k) = P(n > 0 : Xn = N | X0 = k). Since 0 and N are absorbing, we have
f (0) = 0, f (N) = 1. For k {1, . . . , N 1}, we have
f (k) = k f (k 1) + k f (k + 1) = f (k + 1) f (k) =

k
( f (k) f (k 1)).
k

For k {1, . . . , N}, define k = f (k) f (k 1). We can thus rewrite what we have obtained above as
k+1 = kk k ; iterating this we get
k =

1 k1
1
1 k1

k>2

We can now find 1 :


1 = f (N) f (0) = 1 + + N = 1 +
= 1 = 1 +

!
1 k1
1
1
+ +
1
1 k1
!1

1 k1
1
+ +
1
1 k1

Then, for k {1, . . . , N}, we have


k1
1 + 11 + + 11
1
k1
f (k) = f (k) f (0) = 1 + k = 1 + 1 + k = 1
1 N1 .
1
+

25

N1

Series 6: hitting times

Exercise 1
Let (Xn ) be an irreducible Markov chain with stationary distribution . Find E (min{n : Xn = X0 }).
Exercise 2
Let P be the transition matrix of an irreducible Markov chain in state space S . A distribution on S is
called reversible for P if, for each x, y S , we have (x)P(x, y) = (y)P(y, x).
(a) Show that, if is reversible for P, then is stationary for P.
(b) Assuming that is reversible for P, show that, for any n and any x0 , x1 , . . . xn S ,
P (X0 = x0 , . . . , Xn = xn ) = P (Xn = x0 , . . . , X0 = xn ).
(c) Random walk on a graph. Let G = (V, E) be a finite connected graph. For each x V, let d(x)
denote the degree of x. Let P be the Markov transition matrix for the Markov chain on V given by
P(x, y) =

1
d(x)

if (x, y) E;
otherwise.

This is called the random walk on G. Show that (x) = d(x)/

z d(z)

is reversible for P.

Exercise 3
We identify the square grid = {1, . . . , 8}2 with a chessboard. In chess, a knight is allowed to move on
the board as follows. If the knight is in position (x, y), it can go to any of the positions in
!
(x + 1, y + 2), (x 1, y + 2), (x 2, y + 1), (x 2, y 1),

.
(x 1, y 2), (x + 1, y 2), (x + 2, y 1), (x + 2, y + 1)
Let (Xn ) denote the sequence of positions of a knight that at time zero is in position (1, 1) and at each step
chooses uniformly among its allowable displacements. Find the expected time until the knight returns to
(1, 1).
(Hint: describe the chain as in Exercise 2(c).)
Exercise 4
Consider a finite population (of fixed size N) of individuals of possible types
A and a undergoing the following growth process. At instants of time t1 < t2 < t3 < . . ., one individual
dies and is replaced by another of type A or a. If just before a replacement time tn there are j As and
N j as present, we postulate that the probability that an A individual dies is j1 /B j and that an a
individual dies is (N j)2 /B j where B j = 1 j + 2 (N j). The rationale of this model is predicated on
the following structure: Generally a type A individual has chance 1 /(1 + 2 ) of dying at each epoch tn
and an a individual has chance 2 /(1 + 2 ) of dying at time tn . (1 /2 can be interpreted as the selective
advantage of A types over a types). Taking account of the sizes of the population it is plausible to assign
the probabilities 1 j/B j and (2 (N j)/B j) to the events that the replaced individual is of type A and type
a, respectively. We assume no difference in the birth pattern of the two types and so the new individual
is taken to be A with probability j/N and a with probability (N j)/N.
(1) Describe the transition probabilities of the Markov chain (Xn ) representing the number of individuals
of type A at times tn (n = 1, 2, . . .).
(2) Find the probability that the population is eventually all of type a, given k As and (N k) as initially.

26

Exercise 5
Let (Xk ) be a Markov chain for which i is a recurrent state. Show that
lim P(Xk , i for n + 1 6 k 6 n + N | X0 = i) = 0.

If i is a positive recurrent state prove that the convergence in the above equation is uniform with respect
to n.

27

Series 6: hitting times

Solutions

Exercise 1

E (min{n : Xn = X0 }) =

(x) E x (min{n : Xn = X0 }) =

xS

(x)

xS

1
= #S .
(x)

Exercise 2
(a) For any x S ,
X

(y)P(y, x) =

X
y

(x)P(x, y) = (x)

P(x, y) = (x),

so is stationary.
(b)
P (X0 = x0 , . . . , Xn = xn ) = (x0 )P(x0 , x1 )P(x1 , x2 ) P(xn1 , xn )
= P(x1 , x0 )(x1 )P(x1 , x2 ) P(xn1 , xn )
= P(x1 , x0 )P(x2 , x1 )(x2 )P(x2 , x3 ) P(xn1 , xn )
= = P(x1 , x0 )P(x2 , x1 )P(x3 , x2 ) P(xn , xn1 )(xn )
= P (Xn = x0 , . . . , X0 = xn ).
(c) Let x, y V. If x and y are not adjacent, we have (x)P(x, y) = (y)P(y, x) = 0. Otherwise,
1
d(y)
1
d(x)
1

=P

(x)P(x, y) = P
=P
= (y)P(y, x).
d(z)
d(z)
d(z)
d(x)
d(y)
z
z
z

Exercise 3
We give a graph structure to the chessboard by taking the vertex set to be and setting edges x y if
and only if the knight can go from x to y (this implies that it can also go from y to x, so the relation is
symmetric). The stochastic sequence (Xn ) is then a random walk on this graph.
We claim that the graph is connected (equivalently, that the chain is irreducible). To see this, consider the
reduced board = {1, 2, 3}2 . Without leaving , it is possible to go from (1, 1) to (1, 2): performing the
moves (1, 1) (3, 2) (1, 3) (2, 1); of course, reversing these moves allows us to go from (2, 1) to
(1, 1). Now, consider two adjacent positions x, y (x and y differ by 1 in one coordinate and coincide
in the other). For any such pair x, y, it is possible to find a 3x3 subset of such that either x or y is one
of the corners of the square. Then, except possibly for a rotation, the moves given for can be repeated
so that we can go from x to y and from y to x. Now, for arbitrary points x, y , we can construct a path
from x to y such that at each step we go to an adjacent position, so all cases are now covered.
P
By Exercise 2(c), the stationary distribution is given by (x) = d(x)/ z d(z). The degrees of the vertices
28

of the graph are given by


2
3
4
4
4
4
3
2

3
4
6
6
6
6
4
3

4
6
8
8
8
8
6
4

4
6
8
8
8
8
6
4

4
6
8
8
8
8
6
4

4
6
8
8
8
8
6
4

3 2
4 3
6 4
6 4
6 4
6 4
4 3
3 2
P
The expected return time to (1, 1) is given by 1/((1, 1)) = ( z d(z))/d((1, 1)) = 336/2 = 168.
Exercise 4
(1) The transition probabilities are given by
P j, j1 =

1 j(N j)
,
B jN

P j j = 1 P j, j1 P j, j+1 ,

P j, j+1 =

Pi j = 0,

2 (N j) j
,
B jN

for |i j| > 1.

(2) We are exactly in the context of last weeks Exercise 5. Using what was obtained there, we have
P(n : Xn = 0 | X0 = 0) = 1, P(n : Xn = 0 | X0 = N) = 0 and
P(n : Xn = N | X0 = k) =
Since in our present case we have

Pk,k1
Pk,k+1

1
2

1+
1+

P1,0 P2,1
P1,0
P1,2 + P1,2 P2,3
P1,0 P2,1
P1,0
P1,2 + P1,2 P2,3

+
+

P1,0 Pk1,k2
P1,2 Pk1,k
P1,0 PN1,N2
P1,2 PN1,N

for all k, we conclude that


1+

1
2

+ +

1+

1
2

+ +

P(n : Xn = 0 | X0 = k) = 1 P(n : Xn = N | X0 = k) = 1

 k1
1

 N1 =
1

 N
1

 N
1

 k
1

Exercise 5
Let i = min{k > 1 : Xk = i} be the first return time to i. When i is recurrent, we have Pi (i < ) = 1, so
lim Pi (i > K) = 0.

(1)

If, in addition, i is positive recurrent, we have

Pi (i > l) = Ei (i ) < ,

l=0

so that
(2)

lim

Pi (i > l) = 0.

l=K

With this at hand, we are ready:


Pi (k [n, n + N] : Xk = i) =

n1
X

Pi (Xt = i, Xt+1 , . . . , Xn+N , i)

t=0

n1
X

Pi (Xt = i) Pi (i > N + n t) 6

t=0

n1
X
t=0

29

Pi (i > N + n t) =

n
X
l=1

Pi (i > N + l).

It follows from (1) that, if we keep n fixed and take N to infinity, the last expression converges to zero.
In addition, if i is positive recurrent, we can use (2) to obtain
n
X

Pi (i > N + l) =

l=1

N+n
X
l=N+1

as N , so the convergence is uniform in n.

30

Pi (i > l) 6

X
l=N+1

Pi (i > l) 0

Series 7: recap on Markov chains

Exercise 1
Let S = {0, 1, 2, 3}. We consider the Markov chain on S whose transition matrix is given by

1/4 1/4 1/2 0


1/6 1/6 1/3 1/3
.
P =
0 1/3 2/3
0

0
0 1/2 1/2
1) What are the communication classes, the transient states ? Is the chain aperiodic ? If not, give the
period.
2) What is P200 ? What approximately is P100 ?
3) Starting from 2, what is the expected number of visits to state 3 before returning to 2 ?
Exercise 2
Consider the Markov chain on the state space below, where at each step a neighbour is uniformly chosen
among sites that are linked by an edge to the current position.

1) What are the communication classes, the transient states ? Is the chain aperiodic ? If not, give the
period.
2) Is there an invariant measure ? Is it unique ? In case it is, give it.
3) Starting from x, what is the probability to hit S before F ?
4) Starting from x, what is the expected number of visits to F before returning to x ?
Exercise 3
We have n coins C1 , . . . , Cn . For each k, Ck is biased so that it has probability 1/(2k + 1) of falling heads.
We toss the n coins. What is the probability of obtaining an odd number of heads ?
(Hint: letting pk = 1/(2k + 1) and qk = 1 pk , you may study the function
f (x) = (q1 + p1 x) (qn + pn x)
and its power series.)

31

Series 7: recap on Markov chains

Solutions

Exercise 1
The communication classes are {0, 1} and {2, 3}. To see that state 0 is transient, note that for the chain
starting at 0, there is a non-zero probability that the chain goes to state 2. Once there, there is no path
going back to 0, so indeed, the probability that the return time to 0 is infinite is strictly positive. The
same holds for state 1. The chain is aperiodic on its recurrent class {2, 3}.
2) We have
1
5
1
+
= .
P200 = (P00 )2 + P01 P10 =
16 24 48
To approximate P100 (that is, to find what Pn looks like when n tends to infinity), we first compute the
stationary distribution on the communication class {2, 3}, which has transition matrix
"
#
1/3 2/3
.
1/2 1/2
We find a stationary distribution with weights (3/7 4/7). On the set {2, 3}, the chain is irreducible
and aperiodic, so starting from any point, the distribution of Xn converges to the stationary distribution.
Hence, for i, j {2, 3}, one has Pnij ( j) as n tends to infinity. Clearly, if i {2, 3} and j {0, 1}, then
Pnij = 0.
If j {0, 1}, then it is a transient state, so for any i, we have Pnij 0 (in fact, we proved a stronger
P
statement in exercise 1 of series 3: that n Pnij is finite). There remains to determine Pnij when i {0, 1}
and j {2, 3}. Intuitively, the chain enters the class {2, 3} at some point, and then converges to equilibrium
in the class {2, 3}, so we should have
lim Pnij = ( j).

(1)

n+

Here is a rigorous proof of this fact. Let


= inf{n : Xn {2, 3}}.
Since i is transient, Pi [ = +] = 0. For any > 0, we can find N large enough such that Pi [ N] .
For j {2, 3} and n N, we have
(2)

Pnij = Pi [Xn = j] =

N1
X
k=0

We decompose Pi [Xn = j, = k] into

Pi [Xn = j, = k] + Pi [ N] .
| {z }

Pi [Xn = j, = k, X = 2] + Pi [Xn = j, = k, X = 3],


and we apply the strong Markov property at time on each term. The first term becomes
Pi [ = k, X = 2] P2 [Xnk = j] Pi [ = k, X = 2] ( j),
n+

where we used the fact that the Markov chain on {2, 3} is aperiodic. Similarly,
Pi [Xn = j, = k, X = 3] Pi [ = k, X = 3] ( j),
n+

32

and putting the two together, we obtain


Pi [Xn = j, = k] Pi [ = k] ( j).
n+

So the sum appearing in the right-hand side of (2) is such that


N1
X

Pi [Xn = j, = k] Pi [ < N] ( j),


n+

k=0

and 1 Pi [ < N] 1. We have thus proved that


lim sup Pnij ( j) + ,
n+

and
lim inf Pnij (1 )( j).
n+

Since > 0 was arbitrary, this proves (1).


Conclusion :
P100

0 0 3/7
0 0 3/7
lim Pn =
n+
0 0 3/7
0 0 3/7

4/7
4/7
4/7
4/7

3) We can use the fact that this expected value is exactly (3)/(2) = 4/3. We can also take a more
explicit approach and decompose the possible movements of the chain: with probability 1/3 the walk
stays at 2 and therefore the number of visits to 3 is zero. Else it goes to 3, and then stays for a time
geometric(1/2), and then goes back to state 2. The expectation we are looking for is thus
+

2X 1
1
0+
k
3
3 k=1 2

!k

4
.
3

Exercise 2
1) The Markov chain is a random walk on a graph (see exercise 2 of series 5). Let G = (V, E) denote
the graph. G is connected, that is, for any x, y V, there exists a path x0 = x, x1 , x2 , . . . , xN = y such
that (xi , xi+1 ) E for each i. We then have P x (y is ever reached) > P(x, x1 )P(x1 , x2 ) P(xN1 , y) > 0,
so the chain is irreducible, and there are no transient states. The chain is also aperiodic. To see this, fix
any vertex that is the extreme point of one of the diagonal segments. Starting from this vertex, we can
return to it in 3 steps (using the diagonal) and also in 4 steps (not using the diagonal), so the period of
this vertex must divide 3 and 4, so it is 1. Since the chain is irreducible, we conclude that all vertices
have period 1, so the chain is aperiodic.
2) since the chain is irreducible and the state space is finite, there exists a unique invariant measure. We
can look for a reversible measure as in exercise 2 of series 5, and we find that the equilibrium distribution
evaluated at a vertex x is equal to the degree of x divided by the sum of the degrees of all vertices. In
our case, this gives the following invariant measure.
2/52
3/52
3/52
2/52

3/52
5/52
5/52
3/52

3/52
5/52
5/52
3/52

2/52
3/52
3/52
2/52

(Remark: the invariant measure is represented here as a matrix just for ease of notation, following the
positions of the graph; we can of course enumerate the 16 states of the chain and present the invariant
measure as a row vector as we usually do.)

33

3) Consider the diagram


1/2 1 b 1 a
0
b
1/2 1 c 1 a
a
c
1/2 1 b
1
a
b
1/2
The value at each site represents the probability, starting from this site, to arrive at S before F. Notice
that we have used symmetry several times. Using the relation
P x (S is reached before F) =

1 X
Py (S is reached before F),
deg(x) yx

valid for any x < {S , F}, we obtain the system of linear equations

a = 31 + 13 b + 13 c

b = 31 a + 13

c = 2 a + 2 1 + 1 (1 c)
5

n
X

ak xk .

Solving this gives b = 47 .


4) This is (F)/(x) = 2/3.
Exercise 3
Let us write

f (x) =

k=0

By developping, we observe that ak is the probability of getting exactly k heads. We want to know what
is a1 + a3 + a5 + . Note that
n
X
f (1) =
ak ,
k=0

while
f (1) =

n
X
k=0

ak (1)k =

k even

ak

ak .

k odd

So the quantity we are interested in is


f (1) f (1)
.
2
Clearly, f (1) = 1, while an easy computation gives f (1) = 1/(2n + 1). The probability we are interested
in is thus n/(2n + 1).

34

Series 8: Poisson processes

Exercise 1
Let X1 and X2 be independent exponentially distributed random
variables with parameters 1 and 2 so that
P(Xi > t) = exp(i t)
Let
N=

for t > 0.

1 if X1 < X2 ;
2 if X2 6 X1 ,

U = min{X1 , X2 } = XN ,
V = max{X1 , X2 },
W = V U = |X1 X2 |.
Show
a) P(N = 1) = 1 /(1 + 2 ) and P(N = 2) = 2 /(1 + 2 ).
b) P(U > t) = exp{(1 + 2 )t} for t > 0.
c) N and U are independent random variables.
d) P(W > t|N = 1) = exp{2 t} and P(W > t|N = 2) = exp{1 t} for t > 0.
e) U and W = V U are independent random variables.
Exercise 2
Assume a device fails when a cumulative effect of k shocks occur.
If the shocks happen according to a Poisson process with parameter , find the density function for the
life T of the device
Answer:
k k1 t

if t > 0;
t(k)e
f (t) =

0,
if t 6 0.
Exercise 3
Let {X(t), t > 0} be a Poisson process with intensity parameter .
Suppose each arrival is registered with probability p, independently of other arrivals. Let {Y(t), t > 0}
be the process of registered arrivals. Prove that Y(t) is a Poisson process with parameter p.
Exercise 4
At time zero, a single bacterium in a dish divides into two bacteria. This species of bacteria has the
following property: after a bacterium B divides into two new bacteria B1 and B2 , the subsequent length
of time until each Bi divides is an exponential random variable of rate = 1, independently of everything
else happening in the dish.
a) Compute the expectation of the time T n at which the number of bacteria reaches n.
b) Compute the variance of T n .

35

Series 8: Poisson processes

Solutions

Exercise 1
a)
P(N = 1) = P(X1 < X2 ) =

2 e2 y 1 e1 x dy dx =

1
;
1 + 2

2
.
P(N = 2) = 1 P(N = 1) =
1 + 2
b)
P(U > t) = P(X1 , X2 > t) = P(X1 > t) P(X2 > t) = e(1 +2 )t .
c)
Z

1
= P(U > t) P(N = 1).
1 + 2
t
x
P(U > t, N = 2) = P(U > t) P(U > t, N = 1) = P(U > t)(1 P(N = 1)) = P(U > t) P(N = 2).
P(U > t, N = 1) = P(t < X1 < X2 ) =

2 e2 y 1 e1 x dy dx =

d)
P(W > t, N = 1) P(X2 > X1 + t)
=
P(N = 1)
P(N = 1)
Z Z
1
1
1 + 2

e2 t = e2 t .
=
2 e2 y 1 e1 x dydx =
1

1
1
2
0
x+t

P(W > t | N = 1) =

1 +2

Exchanging the roles of X1 and X2 , we get P(W > t | N = 2) = e1 t .


e) For all s, t > 0,
P(U > s, W > t) = P(s < X1 < X2 t) + P(s < X2 < X1 t)
Z Z
Z Z
2 y
1 x
1 e1 x 2 e2 y dx dy
2 e
2 e
dy dx +
=
s

x+t

y+t

1
2
=
e2 t e(1 +2 )s +
e1 t e(1 +2 )s .
1 + 2
1 + 2
Also,
P(W > t) = P(W > t | N = 1) P(N = 1) + P(W > t | N = 2) P(N = 2) =

1
2
e2 t +
e1 t .
1 + 2
1 + 2

We thus have P(U > s, W > t) = P(U > s) P(W > t), so they are independent.
Exercise 2
If T 1 , T 2 , T 3 , . . . are the arrival times for the Poisson process, then T 1 , T 2 T 1 , T 3 T 2 , . . . are independent
and all have exponential() distribution. We are looking for the law of T k = T 1 +(T 2 T 1 )+ +(T k T k1 ).
This is given by the k-convolution of exponential . We claim that the density f (k) of this convolution is
given by
k k1

et if t > 0;
(k)
(k)
f (x) =

0,
otherwise.
36

This is easy to prove by induction: when k = 1, it is trivial; assuming it is true for k, we get
f (k+1) (x) = ( f (k) f (1) )(x) =

e(xy)
0

k+1 x xk
k+1 xk
k yk1 y
e
dy =
e
=
ex
(k)
(k)
k
(k + 1)

since k(k) = (k + 1).


Exercise 3
Let T 1 , T 2 , . . . denote the sequence of arrival times of X(t). One rigorous way of stating what is said in
the exercise is as follows. Let Z1 , Z2 , . . . be a sequence of independent Bernoulli(p) random variables.
PX(t)
Assume that they are also independent of the process X. Then, put Y(t) = i=1
Zi . With this construction,
Zi is 1 if arrival i is registered and 0 otherwise.
Given an interval A [0, ), we will denote respectively by NA and N A the number of arrivals of X(t)
and Y(t) in A. We prove that Y(t) is a Poisson process of parameter p by verifying:
If A, B are disjoint intervals, then N A and N B are independent.
X
P(N A = m, N B = n, NA = m1 , NB = n1 )
P(N A = m N B = n) =
m1 >m
n1 >n

N A = m, N B = n,
NA = m1 , T r , . . . , T r+m1 1 A, NB = n1 , T s , . . . , T s+m2 1 B

!
P 2
Zi = m, s+m
j=s Z j = n,
NA = m1 , T r , . . . , T r+m1 1 A, NB = n1 , T s , . . . , T s+m2 1 B

r,s:
m1 >m,
n1 >n r+m1 6s

r,s:
m1 >m,
n1 >n r+m1 6s

Pr+m1
i=r

NA = m1 , T r , . . . , T r+m1 1 A,
NB = n1 , T s , . . . , T s+m2 1 B

P(Bin(m1 , p) = m) P(Bin(n1 , p) = n)

P(Bin(m1 , p) = m) P(Bin(n1 , p) = n) P(NA = m1 , NB = n1 )

r,s:
r+m1 6s

m1 >m,
n1 >n

m1 >m,
n1 >n

P(Bin(m1 , p) = m) P(NA = m1 )
=
m1 >m

P(Bin(n
,
p)
=
n)

P(N
=
n
)
1
B
1

n1 >n

Repeating the above computation in each of the last big parenthesis, we get
P(N A = m, N B = n) = P(N A = m) P(N B = n)
as required.

P(N [t,t+h] > 2) = o(h). This is true because {N [t,t+h] > 2} {N[t,t+h] > 2} and P(N[t,t+h] > 2) = o(h).
P(N [t,t+h] = 1) = ph + o(h). Indeed,
P(N [t,t+h] = 1) = P(N [t,t+h] = 1, N[t,t+h] = 1) + P(N [t,t+h] = 1, N[t,t+h] > 2)

X
=
P(N[t,t+h] = 1, T i [t, t + h], Zi = 1) + o(h)
i=1

=p

P(N[t,t+h] = 1, T i [t, t + h]) + o(h)

i=1

= p P(N[t,t+h] = 1) + o(t) = ph + po(h) + o(h).

37

Exercise 4
Let R0 = 0 and let R1 < R2 < . . . be the times at which the population size increases. Since we already
start with two bacteria at time 0, we have T n = Rn2 .
Consider the following alternate experiment. Start at time S 0 = 0 a population of 2 bacteria and wait
until one of them reproduces; call this time S 1 . At this instant, we forget about these bacteria and start
watching another population with three new-born bacteria and wait until one of them reproduces; call
this time S 2 . Then yet again we forget about this population and start a new one with four new-born
bacteira, and wait until one of them reproduces, time S 3 , and so on.
By the lack of memory of the exponential distribution, the law of R1 , R2 R1 , R3 R2 , . . . is equal to that
of S 1 , S 2 S 1 , S 3 S 2 , . . . In the alternate experiment it is easy to see that S m S m1 is equal to the
minimum of m + 1 independent exponential(1) random variables (the times until the reproduction of each
bacterium in the mth population) and using Exercise 1b we conclude that S m S m1 has exponential(m+1)
distribution. Thus,
E(T n ) = E(Rn2 ) = E(R1 + (R2 R1 ) + + (Rn2 Rn3 )) =

1
1 1
+ + ... +
.
2 3
n1

Using independence,
Var(T n ) = Var(Rn2 ) = Var(R1 ) + Var(R2 R1 ) + + Var(Rn2 Rn3 )) =

38

1
1
1
+ 2 + ... +
.
2
2
3
(n 1)2

Series 9: More on Poisson processes

Exercise 1
Messages arrive at a telegraph office in accordance with the laws
of a Poisson process with mean rate of 3 messages per hour.
(a) What is the probability that no message will have arrived during the morning hours (8 to 12)?
(b) What is the distribution of the time at which the first afternoon message arrives?
Exercise 2
A continuous time Markov chain has two states labeled 0 and 1.
The waiting time in state 0 is exponentially distributed with parameter > 0. The waiting time in state
1 follows an exponential distribution with parameters > 0. Compute the probability P00 (t) of being in
state 0 at time t starting at time 0 in state 0.

+ +
e(+)t .
Solution: P00 (t) = +
Exercise 3
In the above problem, let = and define N(t) to be the number
of times the system has changed states in time t > 0. Find the probability distribution of N(t).
n
Solution: P(N(t) = n) = et (t)
n! .
Exercise 4
Let X(t) be a pure birth continuous time Markov chain. Assume that
P(an event happens in (t, t + h) | X(t) is odd) = 1 h + o(h);
P(an event happens in (t, t + h)) | X(t) is even) = 2 h + o(h),
where o(h)/h 0 as h 0. Take X(0) = 0. Find the probabilities:
P1 (t) = P(X(t) is odd),

P2 (t) = P(X(t) is even).

Exercise 5
Under the conditions of the above problem, show that
EX(t) =

(1 2 )2
21 2
[exp{(1 + 2 )t} 1].
t+
1 + 2
(1 + 2 )2

39

Series 9: More on Poisson processes

Solutions

Exercise 1
The distribution of the number of messages that arrive during a four-hour period is Poisson(4 ) =
Poisson(12). Thus,
P(No messages from hour 8 to 12) = P(Poi(12) = 0) = e12 .
By the lack of memory of the Poisson process, the arrival time T of the first message after hour 12 is
given by T = 12 + X, where X is a random variable with exponential(3) distribution.
Exercise 2
The forward Kolmogorov equation tells us that
P00 (t) = P00 (t) + P01 (t)
= P00 (t) + (1 P00 (t))
= ( + )P00 (t) + .
We can then observe that

d 
P00 (t)e(+)t = e(+)t .
dt

Hence, there exists a constant c

R such that

P00 (t)e(+)t =

(+)t
e
+ c.
+

Observing further that P00 (t) = 1, we arrive at the conclusion.


Exercise 3
Let T 0 = 0 and T 1 < T 2 < be the times at which the chain changes state. Then, the hypothesis
implies that T 1 , T 2 T 1 , T 3 T 2 , . . . is a sequence of independent random variables with the exponential
distribution. Thus, N(t) = max{n : T n 6 t} is a Poisson process of intensity . In particular, for fixed t
the law of N(t) is Poisson(t), that is, P(N(t) = n) = ((t)n /n!)et .
Exercise 4
For t > 0, h > 0,
P1 (t) = P(X(t + h) is odd) = P (X(t) is odd, no arrivals in [t, t + h])
+ P (X(t) is even, one arrival in [t, t + h])
+ P (X(t + h) is odd, two or more arrivals in [t, t + h])
= P(X(t) is odd) P(No arrivals in [t, t + h]] | X(t) is odd)
+ P(X(t) is even) P(One arrival in [t, t + h] | X(t) is even) + o(h)
= P1 (t)(1 1 h o(h)) + P2 (t)(2 h + o(h)) + o(h).
Dividing by 1/h and taking h to zero, we conclude that P1 is the solution of the differential equation
P1 (t) = 1 P1 (t) + 2 P2 (t).

40

The initial condition is P1 (0) = 0, since we start at zero which is even. The solution of this equation is
P1 (t) =

2
2

et(1 +2 ) .
1 + 2 1 + 2

We then immediately get


P2 (t) = 1 P1 (t) =

2
1
+
et(1 +2 )
1 + 2 1 + 2

Exercise 5
To begin with, note that, for t, h 0, one has
E[X(t + h) X(t)] = P[X(t + h) X(t) = 1] + E[(X(t + h) X(t)) 1X(t+h)X(t)2 ].
We would like to let h tend to 0 in the equality above, and to show that the last term is negligible, that is,
E[(X(t + h) X(t)) 1X(t+h)X(t)2 ] = o(h).
Let = min(1 , 2 ). It is possible to find a coupling of X(t + h) X(t) with a Poisson random variable
N of parameter h, such that X(t + h) X(t) N. So
E[(X(t + h) X(t)) 1X(t+h)X(t)2 ]

+
X

k=2
+
X

kP[X(t + h) X(t) = k]
kP[N = k]

k=2

+
X
k=2

ke h

( h)k
= O(h2 ).
k!

As a consequence,
E[X(t + h) X(t)] = P[X(t + h) X(t) = 1] + o(h)
= P[X(t + h) X(t) = 1 | X(t) is odd]P1 (t) + P[X(t + h) X(t) = 1 | X(t) is even] + o(h),
where
P1 (t) = P[X(t) is odd]
and
P2 (t) = P[X(t) is even].
Using the assumptions of the exercise, we get
E[X(t + h) X(t)] = 1 hP1 (t) + 2 hP2 (t) + o(h).
Hence, the function t 7 E[X(t)] is (right-) differentiable, and
d
E[X(t)] = 1 P1 (t) + 2 P2 (t).
dt
Using the results of the previous exercise, together with the fact that E[X(0)] = 0, we get
Z t
E(X(t)) =
[1 P1 (s) + 2 P2 (s)] ds
0

Z t
1 2 22 s(1 +2 )
21 2
21 2
(1 2 )2
[exp{(1 + 2 )t} 1].
=
+
e
t+

ds =
1 + 2
1 + 2
(1 + 2 )2
0 1 + 2

41

Series 10: Renewal theory

Exercise 1
A patient arrives at a doctors office. With probability 1/5 he
receives service immediately, while with probability 4/5 his service is deferred an hour. After an hours
wait again with probability 1/5 his needs are serviced instantly or another delay of an hour is imposed
and so on.
(a) What is the patients waiting time distribution?
(b) What is the distribution of the number of patients who receive service over an 8-hour period assuming
the same procedure is followed independently for every arrival and the arrival pattern is that of a Poisson
process with parameter 1?
Exercise 2
The random lifetime of an item has distribution function F(x).
Show that the mean remaining life of an item of age x is
R
{1 F(t)}dt
.
E(X x | X > x) = x
1 F(x)
Hint. Recall the derivation, that applies to any positive integrable random variable Z,
Z
Z Z x
Z Z
Z
E(Z) =
x FZ (dx) =
1 dt FZ (dx) =
FZ (dx)dt =
(1 FZ (t))dt.
0

Try to do something similar.


Exercise 3
Find P(N(t) > k) in a renewal process having lifetime density
( (x)
e
for x > ;
f (x) =
0
for x 6 .
where > 0 is fixed.
Exercise 4
For a renewal process with distribution F(t) =

Rt
0

xex dx, show that


x

P(Number of renewals in (0, x] is even) = e

X
n=0

x4n+1
x4n
+
(4n)! (4n + 1)!

Hint:
P(Number of renewals in (0, x] is even) =

X
n=0

42

P(Number of renewals in (0, x] = 2n).

Solutions 10

Solutions

Exercise 1
(a) P(Waiting time = k) = (4/5)k1 (1/5), k = 0, 1, 2, . . ..
(b) We will need the following fact. Let X be a Binomial(N, p) random variable, where N is itself a
Poi() random variable. Then, the distribution of X is Poi(p). This can be proved with probability
generating functions:
gBin(n,p) (s) = (sp + 1 p)n ,
gPoi() (s) = e(s1) ;
!

n
X
X
X
n
e
= e(sp+1p) e = ep(s1) = gPoi(p) (s).
gX =
E(s |N = n) P(N = n) =
(1 + sp p)
n!
n=0
n=0
Now, for i = 0, . . . 7, let Ni denote the number of patients that arrive in the time interval (i, i + 1] and Xi
the number of these patients that receive service before hour 8. Since patients arrive as a Poisson process
with parameter 1 and (i, i + 1] has length 1, Ni has Poisson(1) distribution. For fixed i, the probability
that a given patient that arrives in (i, i + 1] receives service before hour 8 is equal to the probability that
he waits 7 i or less hours for service; this is equal to 1 (4/5)7i+1 = 1 (4/5)8i . Thus, Xi has
Binomial(Ni , 1 (4/5)8i ) = Poi(1 (4/5)81 ) distribution. Notice that X0 , . . . X7 are all independent,
because they are each determined from arrivals of a Poisson process in disjoint intervals and patients
waiting times, which are assumed to be independent. Since for the sum of independent Poisson random
variables has Poisson distribution with parameter equal to the sum of the parameters, we have

!8
!7
!1

4
4
4
+1
+ + 1
X0 , . . . X7 Poi 1
.
5
5
5
Exercise 2
Z
Z Z y
1
1

y F(dy) = x +
1 dt F(dy)
P(X > x) x
P(X > x) x
0
!
Z xZ
Z Z
1

F(dy) dt +
F(dy) dt
= x +
P(X > x)
0
x
x
t
! R
Z
(1 F(t))
1
dt.
x(1 F(x)) +
(1 F(t))dt = x
= x +
P(X > x)
1 F(x)
x

E(X x | X > x) = x +

Exercise 3
Let the sequence X1 , X2 , . . . of independent random variables with the assigned density be the sequence
of inter-arrival times of the renewal process. The density in the statement of the exercise is the density
for a random variable obtained by adding to a exponential() random variable. We can thus write
Xi = Yi + , where Yi exp() and i > 0. Then,
(
0,
if t k 6 0;
P(N(t) > k) = P(X1 + + Xk 6 t) = P(Y1 + + Yk 6 t k) =
k) > k), otherwise,
P(N(t
k) > k) = P(Poi() > k).
where N is a Poisson process of parameter . We further remark that P(N(t

43

Exercise 4
Rx
Let F be the distribution function of the renewal times; we have F(x) = 0 yey dy = 1 ex xex
for x > 0. Also let f (x) = xex I{x>0} be the density function of the renewal times. Let f (n) denote the
n-convolution of f ; let us show by induction that
f (n) (x) =

1
x2n1 ex I{x>0} .
(2n 1)!

Indeed, this trivially holds for n = 1 and, assuming it holds for fixed n and x > 0,
Z x
(n+1)
f (x y) f (n) (y) dy
f
(x) =
0
Z x
1
=
(x y)e(xy)
y2n1 ey dy
(2n 1)!
0
( 2n+1
)
x2n+1
x
1
ex

x2(n+1)1 ex .
=
=
(2n 1)!
2n
2n + 1
(2(n + 1) 1)!
Of course, if x < 0 we have f n+1 (x) = 0 and so the induction is complete. Now, let N(x) denote the
number of renewals in [0, x]; we have
Z x
Z x
1
(2n)
y4n1 ey (e(xy) + (x y)e(xy) ) dy
(1 F(x y)) f (y) dy =
P(N(x) = 2n) =
0 (4n 1)!
0
(Z x
)
Z x
Z x
ex
=
y4n1 dy
y4n1 dy + x
y4n dy
(4n 1)! 0
0
0
( 4n
)
)
( 4n
x
x4n+1
x4n+1
x4n+1
x
ex
x
+

+
=e
.
=
(4n 1)! 4n
4n
4n + 1
(4n)! (4n + 1)!
The result now follows by summing this expression over n.

44

Series 11: more on renewal theory

Exercise 1
The weather in a certain locale consists of rainy spells alternating
with spells when the sun shines. Suppose that the number of days of each rainy spell is Poisson distributed
with parameter 2 and a sunny spell is distributed according to an exponential distribution with mean 7
days. Assume that the successive random durations of rainy and sunny spells are statistically independent
variables. In the long run, what is the probability on a given day that it will be raining?
Exercise 2
Determine the distribution of the total life t of the Poisson process.
Exercise 3
Consider a renewal process with non-arithmetic, finite mean distribution of
renewals, and suppose that the excess life t and current life t are independent random variables for all t.
Establish that the process is Poisson.
Hint: use limit theorems on the identity
P[t x, t > y] = P[t x] P[t > y],
to derive a functional equation for
1
v(x) =

(1 F(t)) dt.
x

Exercise 4
Show that the renewal function corresponding to the lifetime density
f (x) = 2 xex ,
is

x>0

1
1
M(t) = t (1 e2t ).
2
4
Hint. Use the uniqueness of the solution of the renewal equation. Here are some shortcuts for the
computations:
Z T
1
et dt = (1 eT );

0
Z T
1
tet dt = 2 (1 eT T eT );

0
!
Z T
1
2 T 2 eT
2 t
T
T
;
t e dt = 3 1 e
T e

0
Z T
1
tet dt = 2 (1 eT + T eT ).

45

Series 11: more on renewal theory

Solutions

Exercise 1
Since we are only interested in the long run result, we can assume that at time 0 we are starting a rainy
spell. For t > 0, define
A(t) = P(It is raining at time t);
a(t) = P(The initial rainy spell is still taking place at time t).
Also let F denote the distribution of the duration of two successive seasons (i.e., the distribution of a sum
of two independent random variables, one with law Poisson(2) and the other with law exponential(1/7)).
We then have the renewal equation
Z t
A(t s) dF(s).
A(t) = a(t) +
0

(Notice that F is neither purely discrete nor purely continuous, so the integral has to be seen as a sum
plus an integral). From the renewal theorem, we have
Z
1
a(s) ds,
lim A(t) =
t
0
where is the expectation
with F, equal to 2 + 7. Denoting by Y the duration of the initial
R associated
P
2
rainy spell, we have 0 a(s) ds =
s=0 P(Y > s) ds = E(Y) = 2. So the final solution is 2+7 .
Exercise 2
Let denote the parameter of the Poisson process.
We have t = t + t , where t = S N(t)t is the residual life and t = t S N(t) is the current life. Moreover,
since the process is Poisson(), t and t are independent, with distributions given by
Ft (x) =

0, if x < 0;
1 ex
if x > 0.

0,
if x < 0;

x
1 e , if 0 6 x < t;
Ft (x) =

1
if x > t.

Notice that the distribution of t does not depend on t, and is the exponential() distribution.
The distribution of t is given by the convolution of the two above distributions. If x < t, we have
Z x
Ft (x s) Ft (ds) =
Ft (x) =
Z0 x
(1 e(xs) ) es ds = 1 ex xex .
0

If x > t, we have to take into account that the distribution of t has a mass of et in point {t}; the
convolution is then given by
Z x
Ft (x) =
Ft (x s) Ft (ds) =
Z0 t
(1 e(xs) ) es ds + (1 e(xt) )et = 1 ex tex .
0

46

Putting together the two cases, the solution can be expressed as


(
0,
if x < 0;
Ft (x) =
1 ex min(t, x) ex if x > 0.
Exercise 3
It follows from the renewal theorem that
lim P[t > y] = v(y).

t+

Note that
P[t x, t > y] = P[tx > y + x],
and using the previous result, we get that this converges to v(x + y) as t tends to infinity. Similarly,
observing the case y = 0, we infer that
lim P[t x] = v(x).

t+

We thus obtain that v(x + y) = v(x)v(y). Since v is monotone, it is a classical exercise to check that there
must exist c, such that
v(x) = ex
(if g = log v, then g(x + y) = g(x) + g(y). . . ) By differentiating, we find
1 F(t) =

x
e ,

so F is the repartition function of an exponential random variable of parameter (> 0) (and = ).


Exercise 4
Let
M(t) = E(#Arrivals until time t);
= 1 t 1 (1 e2t ).
M(t)
2
4

Our objective is to show that M(t) = M(t).


We know that the renewal equation
Z T
A(T t) F(dt)
A(T ) = F(T ) +
0

has a unique solution and this solution is M (see course notes or page 183 of the textbook). So, we only
R
t) F(dt).
) = F(T ) + T M(T
need to check that M(T
0
First note that, for T > 0,
Z T
F(T ) =
2 tet dt = 1 eT T eT .
0

Next,

"

#
1
1
1
2(T
t)
t) F(dt) =
M(T
(T t) + e
2 tet dt
2
4 4
0
0
Z T
Z
Z
Z
3
3 T 2 t
2 T t
2 2T T t
t
= T
te dt
t e dt
te dt + e
te dt.
2
2 0
4 0
4
0
0
RT
t) F(dt) =
Using the shortcuts in the statement of the exercise and simplifying, we get F(T ) + 0 M(T
) as required.
M(T
Z

47

Series 12: branching processes

Exercise 1
Sir Galton is worried about the survival of his family name. He himself has three sons, and estimates that
each of them has probability 1/8 to have no boy, probability 1/2 to have 1 boy, probability 1/4 to have 2
boys, and probability 1/8 to have 3 boys. He thinks that these probabilities will be constant in time, that
the numbers of children his descendants will have are independent random variables, and since he lives
in the xixth century, he believes that only men can pass on their name. According to these assumptions,
what is the probability that his name will become extinct ?
Exercise 2
A server takes 1 minute to serve each client. During the n-th minute, the number Zn of clients that arrive
and get in line for service is a random variable. We assume that these variables are independent and that
P(Zn = 0) = 0.2,

P(Zn = 1) = 0.2,

P(Zn = 2) = 0.6.

The attendant can leave to have a coffee only if there are no clients to serve. What is the probability that
he can ever leave for a coffee?
Exercise 3
P
(i) Show that, for a positive random variable Y with i2 P(Y = i) < , we have
2
Var(Y) = g
Y (1) + E(Y) E(Y) ,

where gY is the probability generating function of Y.


(ii) Let (Xn )n>0 be a branching process (as is usual, assume X0 = 1) and let Z be a random variable
whose distribution is equal to the distribution of the number of descendents in (Xn ). Let m = E(Z). Using
the relation gXn+1 = gXn gZ , show that E(Xn ) = mn .
(iii) Let 2 = Var(Z). Again using gXn+1 = gXn gZ , show that
gXn+1 = (gXn gZ )(gZ )2 + (gXn gZ )g
Z.
(iv) Show by induction that
Var(Xn ) =

48

mn (mn 1) 2

m2 m
n2

if m , 1;
if m = 1.

Series 12: branching processes

Solutions

Exercise 1
Let p be the probability that the descendence of one of Galtons sons dies out. Since the average number
of boys is strictly larger than 1, we know that p > 0. The fixed point equation in our case reads
p=

1
1
1 1
+ p + p2 + p3 .
8 2
4
8

As always, p = 1 is a solution. It is not the one we are looking for, so we can simplify the equation into
p2 + 3p 1 = 0.
The solution we are looking for is

13 3
.
2
Now, the descendences of Galtons three sons are assumed to be independent. The probability that they
all become extinct is thus
3

13 3
,

2
p=

and eternal survival is the complementary event.

Exercise 2
We use a branching process as a model to describe the problem. Let us suppose that there are X0 clients
in line. They constitute generation 0. The direct descendents of a client are those that arrive while that
client is being served. Generation n + 1 is formed of direct descendents of clients of generation n. Thus,
each one of the Xn clients of generation n will be served during a minute during which a certain number
Zi (i = 1, . . . , Xn ) of clients arrive: his direct descendents. Once all clients of generation n have been
served, we find Xn+1 = Z1 + + ZXn new clients in line Xn is thus a branching process.
The probability that the server can have a pause is equal to the probability of extinction of the branching
process, which in turn is given by the smallest solution of the equation
=

3
1 1
+ + 2 ,
5 5
5

namely = 1/3.
Exercise 3
(i) For |s| < 1, we have
g
Y (s)
When

2
n=1 n

X
X
d2 X n
d2 n
s P(Y = n) =
s P(Y = n) =
= 2
n(n 1)sn2 P(Y = n).
2
ds n=0
ds
n=1
n=0

P(Y = n) < , we can take

g
Y (1) = lim gY (s) =
s1

X
n=1

n(n 1)P(Y = n) =

X
n=1

n2 P(Y = n)

X
n=1

nP(Y = n) = E(Y 2 ) E(Y).

Then,
2
Var(Y) = E(Y 2 ) E(Y)2 = g
Y (1) + E(Y) E(Y) .

49

(ii) We know that the generating function of Xn is given by the recursion formula
gXn+1 = gXn gZ ,
so that we get
gXn+1 = (gXn gZ )gZ

(1)
and then

E(Xn+1 ) = gXn+1 (1) = gXn (gZ (1)) gZ (1) = E(Xn )E(Z),


since gZ (1) = 1. Thus, E(Xn ) = mn for each n > 1.
(iii) (iv) We obviously have Var(X0 ) = 0. Deriving equation (1), we get

gXn+1 = (gXn gZ ) gZ + (gXn gZ )g


z = (gXn gZ )(gZ ) + (gXn gZ )gZ .

We then have
Var(Xn+1 ) = gXn+1 (1) + E(Xn+1 ) E(Xn+1 )2

2
= gXn (gZ (1))(gZ (1))2 + gXn (gZ (1))g
Z (1) + E(Xn+1 ) E(Xn+1 )

= (Var(Xn ) + E(Xn2 ) E(Xn ))E(Z)2 + E(Xn )(Var(Z) + E(Z)2 E(Z)) + E(Xn+1 ) E(Xn+1 )2
= m2 Var(Xn ) + m2(n+1) mn+2 + mn 2 + mn+2 mn+1 + mn+1 m2(n+1)

= mn 2 + m2 Var(Xn ).
Thus,

Var(Xn+1 ) = mn 2 + m2 Var(Xn ) = (mn + mn+1 )2 + m4 Var(Xn1 )

(n + 1)
n
n+1
2n 2
2n+2
= = (m + m + + m ) + m
Var(X0 ) =
n
n+1

m (m 1) 2
m1

since Var(X0 ) = 0. The result is now proved.

50

if m = 1
if m , 1

Series 13: branching and point processes

Exercise 1
Suppose that in a branching process, the number of offspring of an
initial particle has a distribution whose generating function is f (s). Each member of the first generation
has a number of offspring whose distribution has generating function g(s). The next generation has
generating function f , the next g, and the functions continue to alternate this way from generation to
generation.
What is the probability of extinction ? Does this probability change if we start the process with the
function g, and then continue to alternate ? Can the process go extinct with probability 1 in one case, but
not in the other ?
Exercise 2
Consider a branching process with initial size N and probability generating function
(s) = q + ps,

p, q > 0,

p + q = 1.

Determine the probability distribution of the time T when the population first becomes extinct.
Exercise 3
Points are thrown on 2 according to a Poisson point process of rate . What is the distribution of the
distance from the origin to the closest point of the point process ?

Exercise 4
Let t 7 X(t) be a Poisson process of rate . What is the distribution of the set of jumps of the process
t 7 X(et ) ?

51

Series 13: branching and point processes

Solutions

Exercise 1
Considering only even generations, one obtains a usual branching process with generating function
f (g(s)). The probability of extinction is thus the smallest p > 0 such that f (g(p)) = p. If we start
with the generating function g instead of f , then the probability of extinction is the smallest p such that
g( f (p)) = p. These two probabilities are different in general. Choose
f (s) =

1 3
+ s,
4 4

g(s) = s2 .
Solving f (g(p)) = p for 0 < p < 1 leads to p = 1/3, while solving g( f (p)) = p for 0 < p < 1 leads to
p = 1/9.
It is however not possible to find an example where one has a non-zero probability of survival in one
case, but not on the other. Recall that this question is decided by comparing the expected number of
descendants to the value 1. Here, if m1 is the expected number of descendants associated to the generating
function f , and m2 is the expected number of descendants associated to the generating function g, then
the expected number of descendants associated to the generating function f (g(s)) is m1 m2 , which is the
same as the value for g( f (s)). This can be proved by representing the process using random variables,
but can also be seen directly on the characteristic function, since the value is
!
!
d
d
f (g(s)) = g (1) f (g(1)) = g (1) f (1) = m1 m2 =
g( f (s)) .
s=1
s=1
ds
ds

Exercise 2
Let us consider the process started with one individual, and let Xn be the size of the population at time n.
The generating function of Xn is (n) (s), and one can show by induction that
(n) (s) = q + pq + + pn1 q + pn s.
The probability that Xn = 0 is thus
(n) (0) = q + pq + + pn1 q =

q(1 pk )
= 1 pk .
1 p

When the population is started with N individuals, the offspring of these individuals are independent,
and thus
P[T N] = P[the offspring of the N individuals are all extinct]
= (1 pk )N .
Exercise 3
The distance d from the origin to the closest point is larger than r if and only if there is not point of the
Poisson process that falls within the ball of radius r centred at the origin. The number of such points
follows a Poisson distribution with parameter r2 . Hence,
P[d > r] = exp(r2 ).

52

It thus follows that d has a probability density 2r exp(r2 )dr.


Exercise 4
For any interval I, let N(I) be the number of jumps of X occuring during I, and N (I) be the number of
jumps of X that occur during I. One can check that
N([eb , ea ]) = N ([a, b]).
As a consequence, it is easy to see that for two disjoint intervals I1 and I2 , the random variables N (I1 )
and N (I2 ) are independent. Moreover, N ([a, b]) follows a Poisson distribution with parameter
a

(e

e )=

ex dx.

The set of jumps of the process t 7 X(et ) thus forms a Poisson point process with intensity measure
ex dx.

53

Series 14: martingales in discrete time

1.
let Xi be a sequence of independent
V (Xi ) = E[(Xi E[Xi ])2 ] = i2 . Show

Sn =

random variables with

E[Xi ] = 0

and

that the sequence

n
X

(Xi2 i2 )

i=1
is a martingale with respect to

F , the ltration generated by the sequence {Xi }.

Proof. (For denition of martingale see Denition 5.2) We rst check the inte-

grability os

Sn .
n
X

E[|Sn |] = E[|

n
n
X
X
Xi2 i2 |] E[
|Xi2 i2 |]
E[|Xi2 i2 |]

i=1

n
X

i=1

E[Xi2 ] + E[i2 ] =

Sn

i=1

2i2 <

i=1

i=1
hence

n
X

is integrable.

To check that

1, 2, . . . , n

are

Fn

Sn

is

Fn

-measurable, just observe that since

Xi

for

i =

measurable, the sum of them is as well.

What remains to prove is the matringale property.

2
2
2
2
|Fn ] n+1
|Fn ] = E[Sn |Fn ] + E[Xn+1
n+1
E[Sn+1 |Fn ] = E[Sn + Xn+1
2
2
2
2
n+1
= E[Sn |Fn ] + n+1
] n+1
= Sn + E[Xn+1
= Sn .

Sn

Hence

is an

F -martingale.

2.
Let

Xi

be IID with

Xi N (0, 1)

for each

and put

Yn =

Pn

i=1

Xi .

Show that

Sn = exp{Yn n2 /2}
is an

Fn -martingale

for every

R.

Proof. (For denition of martingale see Denition 5.2) We rst check the in-

tegrability os

X N (0, 1)

Sn .

Since

Sn 0

and by knowing that

n
X

Xi n2 /2}] = en

i=1

Sn

Fn

for

/2

n
Y

E[eXi ] = 1 < .

i=1

is integrable.

To check that
are

/2

(moment generating function), we get

E[|Sn |] = E[Sn ] = E[exp{


hence

E[ecX ] = ec

Sn

is

Fn

-measurable, observe that since

Xi

for

i = 1, 2, . . . , n
Sn is

measurable, and the exponential of the sum is continuous, so

measurable as well.

54

What remains to prove is the matringale property.

E[Sn+1 |Fn ] = E[exp{

n
X

Xi n2 /2}|Fn ] = E[Sn exp{Xn+1 2 /2}|Fn ]

i=1

= Sn E[exp{Xn+1 2 /2}|Fn ] = Sn .
{z
}
|
=1 (as in 7.1)

Hence

Sn

is an

F -martingale.

3.
Let

Xi

be a sequence of bounded random variables such that

Sn =

n
X

Xi

i=1
is an

F -martingale.

Show that Cov(Xi , Xj )

Proof. By Proposition 5.4 we get that

n+m

=0

for

E[Xi ] = 0

i 6= j .

for

i > 1,

hence for

1n

we have

Cov(Xn , Xn+m )

= E[Xn Xn+m ] E[Xn ] E[Xn+m ] = E[E[Xn Xn+m |Fn ]]


| {z }
=0

= E[Xn E[Xn+m |Fn ]] = E[Xn E[Sn+m Sn+m1 |Fn ]] = 0


where the last equality stems from the fact that
the sequence

Xi

Sn

is an

F -martingale.

Hence

are mutually uncorrelated.

4.
Let

{Mn }

and

{Nn }

be square integrable

F -martingales.

Show that

E[Mn+1 Nn+1 |Fn ] Mn Nn = hM, N in+1 hM, N in

(1)

Proof. (For denition of square integrability see Denition 8.1, for denition of

quadratic variatio and covariation see page 54) The right hand side of equality
(1) yields

hM, N in+1 hM, N in =


=

n
X

E[(Mi+1 Mi )(Ni+1 Ni )|Fi ]

i=0

n1
X

E[(Mi+1 Mi )(Ni+1 Ni )|Fi ]

i=0

= E[(Mn+1 Mn )(Nn+1 Nn )|Fn ]


= E[Mn+1 Nn+1 Mn+1 Nn Mn Nn+1 + Mn Nn |Fn ].

55

Using the martingale property of the two processes


measurability of

Mn

and

Nn

with respect to

Fn ,

{Mn }

and

{Nn },

and the

we get

E[Mn+1 Nn+1 Mn+1 Nn Mn Nn+1 + Mn Nn |Fn ]


= E[Mn+1 Nn+1 |Fn ] E[Mn+1 Nn |Fn ] E[Mn Nn+1 |Fn ] + E[Mn Nn |Fn ]
= E[Mn+1 Nn+1 |Fn ] Nn E[Mn+1 |Fn ] Mn E[Nn+1 |Fn ] + Mn Nn
= E[Mn+1 Nn+1 |Fn ] Nn Mn ,
and the proof is done.

5.
Let

{Mn }

1. Let

and

{Nn }

and

be square integrable

F -martingales.

be real numbers. verify that, for every integer

n 0,

hM + N in = 2 hM in + 2hM, N in + 2 hN in .
2. Derive the Cauchy-Schwarz inequality

|hM, N in |

hM in

p
hN in , n 0.

Proof. (For denition of square integrability see Denition 8.1, for denition of

quadratic variatio and covariation see page 54) 1) By Denition 8.3 we get

hM + N in =

n1
X

E[(Mi+1 + Ni+1 Mi Ni )2 |Fi ]

i=0

n1
X

E[((Mi+1 Mi ) + (Ni+1 Ni ))2 |Fi ]

i=0

n1
X

E[2 (Mi+1 Mi )2 + 2(Mi+1 Mi )(Ni+1 Ni ) + 2 (Ni+1 Ni )2 |Fi ]

i=0

= 2 hM in + 2hM, N in + 2 hN in ,
which is what we wherev set out to prove.
2) It is easily seen that the quadratic variation is alway positive and by using
this observation, combined with the result from the rst part of this exercise,
we get, for any

R,
0 hM N in = hM in 2hM, N in + 2 hN in .

Let

= hM, N in /hN in ,
0 hM in 2hM, N in + 2 hN in
= hM in 2hM, N i2n /hN in + hM, N i2n /hN in = hM in hM, N i2n /hN in

hence

hM, N i2n /hN in hM in


p
p
hM, N in hM in hN in

and the proof is done.

56

6.
Let

{Mn }

and

{Nn }

be square integrable

F -martingales.

Check the following

parallellogram equality,

hM, N in =

1
(hM + N in hM N in ).
4

Proof. Using the result from part 1) of problem 8.2 we get

hM + N in hM N in
= hM in + 2hM, N in + hN in hM in + 2hM, N in hN in = 4hM, N in ,
hence

hM, N in = 14 (hM + N in hM N in ).

7.
Let

{Mn }

{Nn } be two square integrable F -martingales and let and


F -adapted processes. Derive the Cauchy-Schwarz inequality

and

be bounded

|hIM (), IN ()in |

hIM ()in

Proof. By Proposition 9.3 we have that both

tegrable

F -martingales,

hIN ()in , n 0.

IM ()

and

IN ()

are square in-

so the proof is identical to the one given in part 2) of

problem 8.2.

8.
In this problem we look at a simple market with only two assets; a bond and a
stock. The bond price is modelled according to

n B
n
B0
where

r > 1

=
(1 + r)Bn1
=1

for

n = 1, 2, . . . , N

is the constant rate of return for the bond. The stock price is

asumed to be stochastic, with dynamics

where

s>0

=
(1 + Rn )Sn1
=s

for

n = 1, 2, . . . , N

{Rn } is a sequence of IID random variables on (, F, P). Fur{Fn } be the ltration given by Fn = (R1 , . . . , Rn ) n = 1, . . . , N

and

thermore, let
a) When is

Sn
S0

Sn /Bn

a martingale with respect to the ltration

{Fn }?

We now look at portfolios consisting of the bond and the stock.

For every

n = 0, 1, 2, . . . , N let xn and yn be the number of stocks and bonds respectively


bought at time n and held over the period [n, n + 1). furthermore, let
Vn = xn Sn + yn Bn

57

[n, n + 1),

by the value of the portfolio over

and let

V0

be our initial wealth.

The rebalancing of the portfolio is done in the following way.

At every time

n 1,

we observe the value of our old portfolio, composed at time

which at time

is

xn1 Sn + yn1 Bn .

We are allowed to only use this

amount to to rebalance the portfolio at time

n,

i.e.

we are not allowed to

withdraw or add any money to the portfolio. A portfolio with this restriction is
called a

self f inancingportf olio.


{xn , yn } of {Fn }-adapted

as a pair

Formally we dene a self-nancing portfolio


processes such that

xn1 Sn + yn1 Bn = xn Sn + yn Bn , n = 1, . . . , N.
b) Show that if
so is

Vn /Bn ,

Sn /Bn is a martingale with respect to the ltration {Fn }, then


Vn is the portfolio value of any self-nancing portfolio.

where

Finally we look at a type of self-nancing portfolios called arbitrage strategies.


A portfolio

{xn , yn }

is called an arbitrage if we have

V0 = 0
P(VN 0) = 1
P(VN > 0) > 0
for the value process of the portfolio. The idea formalized in an arbitrage portfolio is that with an initial wealth of
at time

we get a non-negative portfolio value

with probability one, i.e. your are certain to make money on your

strategy. We say that a model is arbitrage free if the model permits arbitrage
portfolios.
c) Show that if

Sn /Bn

is a martingale then every self-nancing portfolio is

arbitrage free.
Let

Qn

be a square integrable martingale with respect to the ltration

such that

Qn > 0 a.s.

and

{Fn }

Q0 = 1 a.s..

d) Show that even if Sn /Bn is not a martingale with respect to the ltration
{Fn }, nding a process Qn as dened above such that Sn Qn /Bn , will give that
Vn Qn /Bn is a martingale with respect to the ltration {Fn }, and furthermore,
that Vn is arbitrage free.
Even though the multiplication of the positive martingale

Qn

might seem unim-

portant, we will later in the course see that this is in fact a very special action
which gives us the ability to change measure.

In nancial applications, this

is important since the portfolio pricing theory say that a portfolio should be
priced under a risk neutral measure, a measure where all portfolios, divided by
the bank process

Bn

should be a martingale.

The reason for this is that the

Sn /Bn or Sn Qn /Bn
Qn guarantees
closely related to Qn

theory is based on a no-arbitrage assumption, which hold if


is a martingale as proven in this exercise.
that

Vn

So the existence of

is arbitrage free, and using a change of measure

we may price any portfolio

Vn

consisting of

Sn

Proof. a) Use Denition 5.2 to conclude that

and

Bn

Sn /Bn

in a consistent way.

is an

{Fn }-martingale

if

the process is integrable, measurable and have the martingale property, i.e. that

58

E[Sn+1 /Bn+1 |Fn ] = Sn /Bn .


and the Rn 's are IID we get

Since

Sn /Bn 0

for every

n, Bn

is deterministic

Qn
n
Y
E[1 + Rn ]
i=1 E[1 + Rn ]
Q
=
E[|Sn /Bn |] = E[Sn /Bn ] =
,
n
1+r
(1
+
r)
i=1
i=1
hence

Sn /Bn

is integrable if

Rn

is.

produkt is a continuous mapping,

Since

Sn /Bn

is

(2)

Fn = (R1 , . . . , Rn ), and the the


Fn -measurable. To check the mar-

tingale property, just add a conditioning to (2),

E[Sn+1 /Bn+1 |Fn ] =

Sn E[1 + Rn |Fn ]
Sn 1 + E[Rn |Fn ]
Sn E[1 + Rn+1 |Fn ]
=
=
.
Bn (1 + r)
Bn
1+r
Bn
1+r

To get the martingale property

E[Sn+1 /Bn+1 |Fn ] = Sn /Bn

we must have

1 + E[Rn |Fn ]
= 1,
1+r
or equivalently that

E[Rn+1 |Fn ] = r.

Hence

Sn /Bn

is an

{Fn }-martingale

if

E[Rn+1 |Fn ] = r.
b) Since the denition of a self-nancing portfolio is that

xn1 Sn + yn1 Bn = xn Sn + yn Bn , n = 1, . . . , N.
we get, by the denition of

Vn ,

xn+1 Sn+1 + yn+1 Bn+1


xn Sn+1 + yn Bn+1
Vn+1
|Fn ] = E[
|Fn ] = E[
|Fn ]
Bn+1
Bn+1
Bn+1
Bn+1
Sn+1
|Fn ] + yn
= xn E[
Bn+1
Bn+1

E[

since

{xn , yn }

are

Fn -measurable.

Under the assuption that

Sn /Bn

is an

Fn -

martingale we get

E[
so

Vn /Bn

Vn+1
xn Sn+1 + yn Bn+1
xn Sn + yn Bn
Vn
|Fn ] = E[
|Fn ] =
=
,
Bn+1
Bn+1
Bn
Bn
is an

Fn -martingale

if

Sn /Bn

is.

Vn = xn Sn + yn Bn is
Vn /Bn is a martingale if Sn /Bn is. To check that any self-nancing
Vn is arbitrage free, we must have V0 = x0 S0 + y0 B0 = 0. Let Sn /Bn

c) From b) we have that any self-nancing portfolio


such that
portfolio

be a martingale, then by Proposition 5.4 a) we have

E[Vn+1 /Bn+1 ] = V0 /B0 = V0 = 0.


Assume that

Bn <

P (Vn 0) = 1
n we get

and

P (Vn > 0) > 0.

Since

E[Vn /Bn ] = 0

for any

E[

Vn
Vn
Vn
] = E[
I{Vn =0} ] + E[
I{Vn >0} ] > 0,
Bn
B
B
| n {z
} | n {z
}
=0

59

>0

and

where

I{}

is the indicator function. This is a contradiction to

d) Following the same lines as in b) we get that if


with respect to the ltration

{Fn }

E[Vn /Bn ] = 0,

Vn .

hence there are no arbitrage strategies

and

Vn

Sn Qn /Bn

is a martingale

is self nancing,

E[

Vn+1 Qn+1
xn Sn+1 Qn+1 + yn Bn+1 Qn+1
xn Sn Qn + yn Bn Qn
Vn Qn
|Fn ] = E[
|Fn ] =
=
,
Bn+1
Bn+1
Bn
Bn

so

Vn Qn /Bn

is an

Fn -martingale

if

Sn Qn /Bn

is. And following the same lines

as the proof of c),

E[

Vn+1 Qn+1
V0 Q0
]=
= V0 = 0.
Bn+1
B0

Assume that P (Vn 0) = 1 and P (Vn > 0) > 0.


Qn > 0, Bn < for any n we get

E[

I{}

E[Vn Qn /Bn ] = 0

and

Vn Qn
Vn Qn
Vn Qn
] = E[
I{Vn =0} ] + E[
I{Vn >0} ] > 0,
Bn
Bn
Bn
|
{z
} |
{z
}
=0

where

Since

>0

is the indicator function. This is a contradiction to

hence there are no arbitrage strategies

E[Vn Qn /Bn ] = 0,

Vn .

9.
A coin is tossed

N is known in advance. 1 unit


1 unit with probability p (1/2, 1]
1 p. If we let Xn n = 1, 2, . . . , N be

times, where the number

invested in a coin toss gives the net protof


and the net prot of

with probability

the net prot per unit invested in the

P(Xn = 1) = p

and

nth

coin toss, then,

P(Xn = 1) = 1 p,

and the Xn 's are independent of each other. Let Fn = (X1 , . . . , Xn )and let
Sn , n = 1, 2, . . . , N ne the wealth of the investor at time n. Assume further
that the initial wealth S0 is a given constant. Any non-nergative amount Cn can
be invested in coin toss n + 1, n = 1, . . . , n 1, but we assume that borrowing
money is not allowed so Cn [0, Sn ]. Thus we have

Sn+1 = Sn + Cn Xn+1 , n = 1, . . . , N 1

and

Cn [0, Sn ].

Finally assume that the objective of the investor is to maximize the expected
rate of return
a) Show that

E[(1/N ) log(SN /S0 )].


Sn

is a submartingale with respect to the ltration

{F}.

Cn the investor use in the investment game,


= p log(p) + (1 p) log(1 p) + log(2) is a superto the ltration {Fn }.

b) Show that whatever strategy

Ln = log(Sn ) n

where

martingale with respect

Hint: At some point you need to study the function

g(x) = p log(1 + x) + (1 p) log(1 x)

60

for

x [0, 1]

and

p (1/2, 1).

c) Show that the fact that

log(Sn ) n

is a supermartingale implies that

E[[] log(SN /S0 )] N .

d) Show that if

Cn = Sn (2p 1), Ln

is an

{F}-martingale.

Proof. (For denition of submartingale and supermartingale see the text follow-

ing Denition 5.2) a) To show that


to show that

Sn

is a submartingale w.r.t.

{F}

we want

E[Sn+1 |Fn ] Sn ,

E[Sn+1 |Fn ] = E[Sn + Cn Xn+1 |Fn ] = {Cn

and

= Sn + Cn E[Xn+1 |Fn ] = {Xn+1

Sn

are

Fn -measurable}

Fn }
= Sn + Cn E[Xn+1 ] = Sn + Cn (1 p 1 (1 p)) Sn ,
|{z} |
{z
}
independent of

hence

Sn

is a submartingale w.r.t.

b) We now want to show that

>0

{Fn }.

E[Ln+1 |Fn ] Ln ,

E[Ln+1 |Fn ] = E[log(Sn+1 ) (n + 1)|Fn ] = E[log(Sn + Cn Xn+1 )|Fn ]


(n + 1) = E[log(Sn (1 + Cn Xn+1 /Sn ))|Fn ] (n + 1)
= E[log(Sn ) + log(1 + Cn Xn+1 /Sn ))|Fn ] (n + 1)
= log(Sn ) n +E[log(1 + Cn Xn+1 /Sn ))|Fn ]
|
{z
}
=Ln

= Ln + p log(1 + Cn /Sn ) + (1 p) log(1 Cn /Sn ) = Ln + g(Cn /Sn ) .


{z
}
|
=g(Cn /Sn )

g 00 (x) = p/(1+x2 )(1p)/(1x2 ) < 0 for x [0, 1), g is concave in that


region, and the maximum is x
= 2p1 since g 0 (
x) = p/(1+
x)(1p)/(1
x) = 0
so g(x) g(
x) for all x [0, 1]. Since Cn /Sn [0, 1],
Since

g(Cn /Sn ) g(
x) = g(2p 1) = p log(p) + (1 p) log(1 p) + log 2 = ,
hence

E[Ln+1 |Fn ] = Ln + g(Cn /Sn ) Ln + = Ln ,


so

Ln

is a supermartingale w.r.t.

c) We have just shown that

{Fn }.

{Fn }.

Ln = log(Sn ) n is a
E[Ln ] L0 so

supermartingale w.r.t.

Because of this we also have that

E[log(SN ) N ] log(S0 ) 0 E[log(SN /S0 )] N .

d) For

Cn = SN (2p 1)

we get

E[Ln+1 |Fn ] = Ln + g(Cn /Sn ) = Ln + g(2p 1) = Ln ,


hence

Ln

is a

{Fn }-martingale

61

using the strategy

Cn = Sn (2p 1).

10.

Xn n = 0, 1, 2, . . . is that price of a stock at time n and assume that


{Fn }. This means that if
we buy one unit of stock at time n, paying Xn , the expected price of the stock
tomorrow (represented by the time n + 1) given the information Fn is lower
Assume that

Xn

is a supermartingale with respect to the ltration

than today's price. in other words, we expect the price to go down. Investing in
the stock does not seem to be a good idea, but is it possible to nd a strategy
that performs better? The answer is no, and the objective of this exercise is to
show that. Let

Cn

be a process adapted to

representing our investment strategy.


after

X.

days is given by

IX (C)n ,

with

0 Cn n = 0, 1, 2, . . .,

the sochastic integral of

Now, show that for any supermartingale

bounded process

{Fn }

We know that the gain of our trading

Xn

with respect to

and any positive, adapted and

Cn
E[IX (C)n+1 |Fn ] IX (C),

i.e. that

IX (C)n

is also a supermartingale.

I (C) see Denition 9.1) We


PX
n1
i=0 Ci (Xi+1 Xi ) so taking

Proof. (For denition of the stochastic integral

may write the stochastic integral as

IX (C)n =

the conditional expectation of the stochastic integral we get

n
X
E[IX (C)n+1 |Fn ] = E[
Ci (Xi+1 Xi )|Fn ]
i=0

= E[

n1
X

Ci (Xi+1 Xi ) + Cn (Xn+1 Xn )|Fn ]

i=0

= E[IX (C)n + Cn (Xn+1 Xn )|Fn ] = {IX (C)n

is

= IX (C)n + E[Cn (Xn+1 Xn )|Fn ] = {Cn

and

Xn

Fn

-measurable}

= IX (C)n + Cn (E[Xn+1 |Fn ] Xn ) {Xn

is supermartingale and

are {Fn }-adapted}

IX (C)n + Cn (Xn Xn ) = IX (C)n .


Hence

IX (C)n

is a supermartingale with respect to

62

{Fn }

if

Xn

is.

Cn 0}

Series 15: discrete Brownian motion


1.
Let

Bn , n = 0, 1, 2, . . .

be a discrete Brownian motion. Show that

Bn P
0
hBin
that is for every

as

n ,

>0



Bn


P
> 0
hBin

as

n .

Proof. Recall that for a square integrable random variable

X ,the

Chebyshev's

inequality is

P(|X| > )
Since

E[X 2 ]
.
2

hBin = n, wich is given in the text at page 64 if needed, we get








Bn
Bn
n
E[Bn2 ]




=
P
>

=
P
>

=
P
(|B
|
>
n)

n
n
hBin
(n)2
(n)2
1 1
= 2 0 as n .
 n

Hence

Bn P
0
hBin

as

n .

2.
Assume the value of the bond's rate of return
is

r = e2 1

random variable

for some constant . What should be the distribution of the

(1+Rn ) in order to model Sn = Sn /Bn as a geometric Brownian

motion i.e.
1

2
Sn = seWn 2n , S0 = s,

where

Wn

is a discrete Brownian motion.

Proof. from the denition of

Sn

we get

(1 + Rn )Sn1
(1 + Rn )Sn1
Sn
.
=
=
Sn =
1 2
Bn
(1 + r)Bn1
e 2 Bn1
We get

(1 + Rn+1 )
(1 + Rn+1 )
Sn+1
=
,
=
1 2
1 2

Sn
e2
e2
so letting

Sn

be a geometric Brownian motion, we must have


1

2
1 2
Sn+1
seWn+1 2(n+1)
= se(Wn+1 Wn )2 .
=
1
Sn
seWn 2n2

Combining the two results we get


1 2
Sn+1
(1 + Rn+1 )
,
= se(Wn+1 Wn )2 =
1 2
Sn
e2

which holds if

1 + Rn+1 = se(Wn+1 Wn ) .
63

Series 16: martingales in continuous time

1.
Let

{Mt }

1. Let

and

{Nt }

and

be square integrable

Ft -martingales.

be real numbers. verify that, for every

t 0,

hM + N it = 2 hM it + 2hM, N it + 2 hN it .
2. Derive the Cauchy-Schwarz inequality

|hM, N it |

p
p
hM it hN it , t 0.

Proof. (For denition of square integrability see Denition 11.3, for denition

of quadratic variation and covariation see pages 74-75) 1) We use the denition
of the covariation process to get

hM + N it = lim

n1
X

kk0

= lim

kk0

= lim

kk0

= lim

kk0

+ lim

kk0

(Mi+1 + Ni+1 Mi Ni )2

i=0

n1
X

((Mi+1 Mi ) + (Ni+1 Ni ))2

i=0
n1
X

2 (Mi+1 Mi )2 + 2(Mi+1 Mi )(Ni+1 Ni ) + 2 (Ni+1 Ni )2

i=0
n1
X

2 (Mi+1 Mi )2 + 2 lim

kk0

i=0
n1
X

n1
X

(Mi+1 Mi )(Ni+1 Ni )

i=0

2 (Ni+1 Ni )2

i=0

2 hM it + 2hM, N it + 2 hN it ,
which is what we wherev set out to prove.
2) Recall the Cauchy-Schwarz inequality for

n-dimensional

n
X

v
u n
n
uX X
b2i .
ai bi t
a2i

i=1

i=1

euclidean space

i=1

We have

hM, N it = lim

kk0

n1
X

(Mi+1 Mi )(Ni+1 Ni )

i=0

v
un1
n1
uX
X
(Ni+1 Ni )2
lim t
(Mi+1 Mi )2
kk0

i=0

64

i=0

and since

is continuous the limit may be passed inside the root sign

v
u
n1
n1
u
X
X
hM, N it t lim
(Mi+1 Mi )2
(Ni+1 Ni )2
kk0

i=0

i=0

p
= hM it hN it
and the proof is done.

2.
Let

{Mt }

and

{Nt }

be square integrable

Ft -martingales.

Check the following

parallellogram equality,

hM, N it =

1
(hM + N it hM N it ), t 0.
4

Proof. Using the result from part 1) of problem 11.1 we get

hM + N it hM N it
= hM it + 2hM, N it + hN it hM it + 2hM, N it hN it = 4hM, N it ,
hence

hM, N it = 14 (hM + N it hM N it ).

65

Series 17: martingales in continuous time


1.
(The value of a European Call Option ). In the Black.Scholes model, the price

St

of a risky asset (i.e. an asset that has no deterministic payo )at time

is

given by the formula


1

St = se(r 2
where

Bt

is a Brownian motion and

)t+Bt

is a positive constant representing the

initial value of the asset. The value of a European Call option, with maturity
time

and strike price

is

(ST K)+

at time

T.

If

T > t,

compute explicitly

E[(ST K)+ |Ft ].


Proof. Because of the Markov property of the Brownian motion, any expectation

h of the Brownian motion evaluated at time T , h(BT ), conditioned


t < T is only dependent on the value Bt and the time to maturity

of a function
on a time

T t.

By Proposition 12.4 we get

E[(ST K)+ |Ft ] = E[(St e(r


= {Proposition

12.4}

2
2

)(T t)+(BT Bt )

= EBt [(St e(r

2
2

66

K)+ |Ft ]

)(T t)+(BT Bt )

K)+ ]

BT Bt =

and by the time homogeneity we may write

N (0, 1),

T tX

where

so

E[(ST K)+ |Ft ] = EBt [(St e(r

2
2

)(T t)+(BT Bt )

K)+ ]

= EBt [(St e(r 2 )(T t)+ T tX K)+ ]


Z

2
x2
1
(St e(r 2 )(T t)+ T tx K)+ e 2 dx.
=
2 R
Since

()+

is non-zero only when

rewritten to get

St e(r

2
2

)(T t)+ T tx

which may be

separated as

log

K
St

(r 2 )(T t)

.
T t

Call the right hand side of the inequality

d1 ,

the integral may be written as

x2
2
1
E[(ST K)+ |Ft ] =
(St e(r 2 )(T t)+ T tx K)e 2 dx
2 d1
Z
Z

x2
2
x2
1
1
=
St e(r 2 )(T t)+ T tx e 2 dx K
e 2 dx
2 d1
2 d1
{z
}
|

1
= St er(T t)
2

2
2

(T t)+

2
T tx x2

d1

P(Xd1 )
Z

1
dx K
2
|

d1

{z

P(Xd1 )

x2
2

dx
}

2
1
1
= St er(T t)
e 2 (x T t) dx KP(X d1 )
2
d1

= {y = x T t, dy = dx}
Z
2
1
y2
r(T t)

e
= St e
dy KP(X d1 )
2 d1 T t
{z
}
|

=P(Xd1 T t)

= St er(T t) P(X d1 T t) KP(X d1 ).


This is the explicit form of the Call Option price.

2.
Let

Bt

be a one dimensional Brownian motion and let

generated by

Bt .

Ft

be the ltration

Show that

E[Bt3 |Fs ] = Bs3 + 3(t s)Bs .


Proof. We start by separating the process into a part that is measurable with

respect to

Fs

and one that is independent of

Ft ,

namely

E[Bt3 |Ft ] = E[(Bt Bs + Bs )3 |Ft ] = E[(Bt Bs )3 + 3(Bt Bs )2 Bs


+ 3(Bt Bs )Bs2 + Bs3 |Ft ] = E[(Bt Bs )3 ] + 3Bs E[(Bt Bs )2 ]
+ 3Bs2 E[Bt Bs ] + Bs3 .

67


Bt Bs N (0, t s)

and since the normal distribution is symmetric all odd

moments is zero, so

E[Bt3 |Ft ] = 0 + 3Bs (t s)2 + 0 + Bs3 = Bs3 + 3(t s)Bs .

3.
Show that the following processes are martingales with respect to
tion generated by theone dimensional Brownian motion
1.

Bt3 3tBt

2.

Bt4 6tBt2 + 3t2 .

Ft ,the

ltra-

Bt

Proof. 1) From Exercise 12.2 we have that

E[Bt3 |Ft ] = Bs3 + 3(t s)Bs .


Using this together with the fact that

Bt

is an

Ft -martingale

we get

E[Bt3 3tBt |Ft ] = Bs3 + 3(t s)Bs 3tBs = Bs3 3sBs


Bt3 3tBt with respect to Ft .
4
2) We start by computing E[Bt |Fs ], and as in Exercise 12.2 we do this by
separating Bt in a part that is measurable with respect to Fs and part that is

which proves the martingale property of

independent of

Fs

E[Bt4 |Fs ] = E[(Bt Bs + Bs )4 |Fs ] = E[(Bt Bs )4 + 4(Bt Bs )3 Bs


+ 6(Bt Bs )2 Bs2 + 4(Bt Bs )Bs3 + Bs4 |Fs ] = E[(Bt Bs )4 ]
+ 4Bs E[(Bt Bs )3 ] + 6Bs2 E[(Bt Bs )2 ] + 4Bs3 E[Bt Bs ] + Bs4 .

Recall that Bt Bs N (0,


t s) so we may write Bt Bs = T sX
X N (0, 1) so we may rewrite our expression as

where

3
E[Bt4 |Fs ] = (t s)2 E[X 4 ] + 4Bs t s E[X 3 ] + 6Bs2 (t s)E[X 2 ]

+ 4Bs3 t sE[X] + Bs4


and since all odd moments of the standard normal distribution is zero and the
second moment is one we have

E[Bt4 |Fs ] = (t s)2 E[X 4 ] + 6Bs2 (t s) + Bs4 .


To evaluate

E[X 4 ]

we use the moment generating function of the standard nor-

mal distribution

X (u) = E[euX ] = eu
and use the result that the

n'th

moment of

X.

n'th

/2

X (u) evaluated
X (u) is

derivative of

The fourth derivative of

(n)

X (u) = (3 + 6u2 + u4 )X (u)

68

in

u=0

is the

and since

X (0) = 1

(4)

X (0) = E[X 4 ] = 3.

we get

From this we get

E[Bt4 |Fs ] = (t s)2 E[X 4 ] + 6Bs2 (t s) + Bs4 = 3(t s)2 + 6Bs2 (t s) + Bs4 .
We now may derive the martingale property of

2
that Bt

in an

Bt4 6tBt2 + 3t2 ,

using the fact

Ft -martingale,

E[Bt4 6tBt2 + 3t2 |Fs ] = 3(t s)2 + 6Bs2 (t s) + Bs4 6tE[Bt2 |Fs ] + 3t2
= 3(t s)2 + 6Bs2 (t s) + Bs4 6tE[Bt2 t + t|Fs ] + 3t2
= 3(t s)2 + 6Bs2 (t s) + Bs4 6t(Bs2 s + t) + 3t2
= 3t2 6ts + 3s2 + 6Bs2 t 6sBs2 + Bs4 6tBs2 + 6ts 6t2 + 3t2
= Bs4 6sBs2 + 3s2 ,
hence

Bt4 6tBt2 + 3t2

is an

Ft -martingale

4.

{ti }
i=0 be an increasing
ti+1 . Furthermore, let

Let

ti

sequence of scalars and dene

Sn =

n1
X

ti

such that

ti <

Bti (Bti+1 Bti ),

i=0
where

Bti

is the discrete Brownian motion.

Sk , 0 k n
B.

Check that
generated by

is not a martingale with respect to the ltration

Proof. We only chek the martingale property of

k1
X

E[Sk |Fk1 ] = E[

Sk .

Bti (Bti+1 Bti )|Fk1 ] = {Bti (Bti+1 Bti )

i=0
are

Fk1 -measurable

for

i k 2} =

k2
X

Bti (Bti+1 Bti )

i=0

{z

k1
=S

+ E[Btk1 (Btk Btk1 )|Fk1 ] = {E[Btk1 (Btk Btk1 )|Fk1 ] = 0}


= Sk1 + E[Btk1 (Btk Btk1 )|Fk1 ] E[Btk1 (Btk Btk1 )|Fk1 ]
= Sk1 + E[(Btk1 Btk1 )(Btk Btk1 )|Fk1 ] = Sk1 + (tk1 tk1 )
6= Sk1
for any

tk1 < tk1 tk

hence

Sk

is not a martingale with respect to the

ltration generated by the Brownian motion

69

B.

5.
Let

Bt

Xt

be a Brownian motion and let

be the stochastic integral

est dBs

Xt =
0
1. Determine the expectation

E[Xt ]

and the variance

V (Xt )

of

Xt .

2. Show that the random variable

Wt =
has distribution
Proof. 1) By part

2(t + 1)Xlog(t+1)/2

Wt N (0, t).

(vi)

of Proposition 13.11, that denes properties of then Ito

integral, we have that since the integrand,

est ,

of the stochastic integral is

deterministic, the stochastic integral is normally distributed as


 Z t

 Z t

 1
Xt N 0,
(est )2 ds = N 0,
e2(st) ds = N 0, (1 e2t ) .
2
0
0


1
2t
Hence Xt has the distribution Xt N 0, (1 e
) .
2


1
2t
2) From the rst part of the exercise, we know that Xt N 0, (1 e
)
.
2
For a normally distrbuted random variable

N (0, c2 2 )

Y N (0, )

it holds that

cY

hence




 p
2 1
(1 e2 log(t+1)/2 ) = N 0, (t + 1)(1 e log(t+1) )
Wt N 0,
2(t + 1)
2


 
1 
t 
= N 0, (t + 1)(1
) = N 0, (t + 1)
= N 0, t .
t+1
t+1
And the proof is done.

6.
Let

be a Brownian motion. Find

zR
Z

F () = z +

and

(s, ) V

(s, )dBs
0

in the following cases


1.

F () = BT3 ().

2.

F () =

3.

F () = eT /2 cosh(BT ()) = eT /2 12 (eBT () + eBT () )

RT
0

Bs3 ds.

70

such that

Proof. 1) We get

1
d(Bt3 ) = 3Bt2 dBt + 6Bt dt
2
and since

d(tBt ) = tdBt + Bt dt
Rt
Rt
tB = 0 sdBs + 0 Bs ds which may be rewritten
Rt
Rt t
Bs ds = 0 (t s)dBs . We may now write the BT3
0

we have

using

to get

as

Rt
0

tdBs

BT3

tBt =

3(Bt2 + (T t))dBt

=z+
0

where

z = E[BT3 ] = 0.

Hence

(s, ) = 3(Bs ()2 + (T s))

2)

d(T BT3 ) = T (3BT2 dBT + 3BT dT ) + BT3 dT


hence

T BT3 = z +

s3Bs2 dBs +

z R.
Z

Bs3 ds,

3sBs ds +
0

0
for some

Rewriting the expression gives

Bs3 ds

=z+

T BT3

s3Bs2 dBs

3sBs ds
0

which by problem 1) gives

Bs3 ds = z +

RT

We need to rewrite

Z

3T (Bs2 + (T s)) s3Bs2 dBs

3sBs ds.

ds.

3sBs ds

on a form that is with respect to

dBs

instead of

Study

d(T 2 BT ) = 2T BT dT + T 2 dBT ,
hence

T 2 BT =

RT
0

2sBs ds +

RT
0

s2 dBs

sBs ds =
0

1
2

and by writing

T 2 BT =

RT
0

T 2 dBs

we get

(T 2 s2 )dBs ,

hence

Bs3 ds = z +

We may now write the

Z
0

Bs3 ds

Z

3 T 2
3T (Bs2 + (T s)) s3Bs2 dBs
(T s2 )dBs .
2 0
RT 3
Bs ds as
0

Z
=z+
0


3
3T (Bs2 + (T s)) s3Bs2 (T 2 s2 ) dBs .
2

71

RT
RT
z = E[ 0 Bs3 ds] = 0 E[Bs3 ]ds = 0.
s)) s3Bs ()2 23 (T 2 s2 ).

Hence

where

(s, ) = 3T (Bs ()2 + (T

3) We notice that since

d(eBt t/2 ) = eBt t/2 dBt


d(eBt +t/2 ) = eBt +t/2 dBt ,
we may write

1
1
eT /2 (eBT () + eBT () ) = eT /2 (eT /2 eBT ()T /2 + eT /2 eBT ()+T /2 )
2
2
Z T
Z T
1
Bs s/2
T /2
T /2
T /2
eBs +s/2 dBs )
e
dBs e
=z+e
(e
2
0
0
Z T
Z T
1
= z + (eT
eBs +s/2 dBs )
eBs s/2 dBs
2
0
0
Z T T Bs s/2
e e
eBs +s/2
=z+
dBs
2
0
where

is

1
1
z = E[F ] = eT /2 E[eBT ] + E[eBT ] = eT /2 eT /2 E[eBT T /2 ]
|
{z
}
2
2
=1

+ eT /2 E[eBT +T /2 ]
|
{z
}

 1
1
= eT /2 eT /2 + eT /2 = (eT + 1).
2
2

=1

7.
Let

Xt

be a generalized geometric grownian motion given by

dXt = t Xt dt + t Xt dBt
where

and

(3)

are bounded deterministic functions and

is a Brownian

motion.
1. Find an explicit expression for
2. Find

zR

Proof. 1) Let

and

(t, ) V

Rt

Xt = e

dXt = t e

Rt
0

s ds
s ds

Yt ,

Xt

E[Xt ].
Rt
X(T, ) = z + 0 (s, )dBs ().

and compute

such that

the dierential of

Yt dt + e

Rt
0

s ds

Xt

is

dYt = t Xt dt + e

Rt
0

s ds

dYt

for this expression to be equal to (3) we must have

which holds if

Rt
0

s ds

dYt = Yt dBt

dYt = t e
|

hence

Yt = y0 e

72

Rt
0

s ds

{z

=Xt

Yt dBt
}

Yt is an exponential martingale given by


R
Rt
dBt 12 0t t2 dt
0 t

where

y0 = 1

since

X0 = 1.

This gives the following expresiion for

Xt = e

Rt
0

Using the martiongale property of

E[Xt ] = E[e

Rt

s ds

Xt

R
t dBt + 0t (t 12 t )2 dt

Yt

gives

Yt ] = e

Rt
0

s ds

E[Yt ] = e
| {z }

Rt
0

s ds

=Y0

RT

XT = e 0 s ds YT where YT is the exponential


martingale with SDE dYt = t Yt dBt hence XT may be written as
Z T R
Z T
RT
RT
RT
T
s ds
s ds
s ds
0
0
0
e 0 s ds s Ys dBs
s Ys dBs ) = |e {z } +
XT = e
YT = e
(1 +
2) In part 1) it was shown that

and

(s, ) = e

RT

s ds

s Ys ().

=z

(s, ) V ,

We need to show that

the criterias

are given in Denition 13.1. Part 1 and 2 of Deniton 13.1 is showed by noticing
that

therefore universally measurable, and

RT

u du

s which is deterministic and


Ys which is the exponential martingale and

is the product of the processes

therefore fullll the conditions 1) and 2). The product of these two processes
does also fullll criterias 1) and 2).

by using that

|t | < K

(t e

|e
Yt2 = e

Rt
0

0
RT
0

0<K <

such that

t [0, T ],

for every


E

Condition 3) of Denition 13.1 is shown

is bounded so that there is a constant

RT
0

s ds

RT


Yt ) dt = e 0 s ds E
2


t2 Yt2 dt {|| < K}

(4)

0
s ds


2
{z K } E

=
R
2s dBs 0t s2 ds


Yt2 dt = {Fubini} =

 
E Yt2 dt.

(5)

can be written as the product of an exponential martin-

Yt2 = e

gale and a deterministic function as

Rt
0

2s dBs 12

Rt
0

(2)2s ds

Rt
0

s2 ds

so that

(4) becomes


E

(t e

RT
0

s ds


Yt )2 dt

Rt
Rt 2 
 Rt
2
1
E e 0 2s dBs 2 0 (2s ) ds e 0 s ds dt

Rt

 Rt
2
2
1
e 0 s ds E e 0 2s dBs 2 0 (2s ) ds dt =
Rt

Rt
0

K 2 ds

dt

0
Hence

Rt
0

s2 ds

dt

K2t

dt =

K2t

dt =

eK

K2

< .

s V .

8.
2

f (t) = et /2 1 and let B be a brownian motion on the probability space


such
(, F, {Ft , t t}, P). Show that there exists another Brownian motion B

Let

that

Z
Xt =
0

f (t)

dBs =
1+s

73

Z
0

s
sdB

Xt

Proof. The quadratic variation of

f (t)

Z
hXit =
0

= t2 /2 =

1 

ds =
1+s

is
2 /2

et


2
1
ds = log 1 + (et /21)
1+s

sds.
0

of (, F, {Ft , t
F,
{Ft , t t}, P)
(,
such that
B

By Theorem 15.4 there exists an extension

t}, P)

on which there is a Brownian motion

s
sdB

Xt =
0
which solves the problem at hand.

9.
Let

Xt

solve the SDE

dXt = (Xt + )dt + (Xt + )dBt X0 = 0


, , and are
2
St = e( /2)t+Bt .

where
let

1. derive the SDE satised by


2. Show that

dSt1 =

is a Brownian motion. Furthermore,

St1 .

d(Xt St1 ) = ( )St1 dt + St1 dBt .

3. Derive the explicit form of


Proof. 1) Let

constants and

f (t, x) = e(

Xt .

/2)tx

then the SDE of

St1

is

1 2
f (t, Bt )dt +
f (t, Bt )dBt +
f (t, Bt )dt
t
x
2 x2

= ( 2 /2)e(

/2)tx

dt e(

/2)tx

2
1
dBt + 2 e( /2)tx dt
2

= ( 2 )St1 dt St1 dBt .


2)

1
i = Xt (( 2 )St1 dt St1 dBt )
d(Xt St1 ) = Xt dSt1 + s1
t dXt + hX, S

+ St1 ((Xt + )dt + (Xt + )dBt ) + (St1 )(Xt + )dt


= ( 2 )Xt St1 dt Xt St1 dBt + Xt St1 dt
+ St1 dt + Xt St1 dBt + St1 dBt 2 Xt St1 dt St1 dt



= ( 2 ) + 2 Xt St1 dt + + Xt St1 dBt + St1 dt


+ St1 dBt = ( 2 ) + 2 Xt St1 dt + + Xt St1 dBt


+ St1 dt + St1 dBt = St1 dt + St1 dBt .
3) From 2) we get that

Xt St1

Ss1 ds

Z
+

St

and express

( 2 /2)t+Bt

St

( )
Xt = e
Z t

2
e( /2)sBs dBs .
+
0

74

Ss1 dBs

0
multiplying both sides by

on its explicit form gives

Z
0

e(

/2)sBs

ds

Series 18: Additional exercises

1
Compute the stochastic dierential

2.

Zt = e , R, t [0, ).
Rt
Zt = 0 g(s)dBs where g(s) is

3.

Zt = eBt .

4.

Zt = eXt

5.

Zt = Xt2

1.

dz

when

where

where

an adapted stochastic process.

dXt = dt + dBt .

dXt = Xt dt + Xt dBt .

t is of rst variation, dZt = et dt = Zt dt.


dZt = g(t)dBt since it is the dierential of an integral.
x
Let f (x) = e
then, using the Ito formula, since Bt

Proof. 1) Since

2)
3)

has quadratic

variation, we get

1
2 Bt
dZt = f 0 (Bt )dBt + f 00 (Bt )dhBit = eBt dBt +
dt
e
2
2
2
= Zt dBt +
Zt dt.
2
4) Using the same notation as in 3) we get

2 Xt 2
1
dt
dZt = f 0 (Xt )dXt + f 00 (Xt )dhXit = eXt (dt + dBt ) +
e
2
2
2
()
= Zt dBt + ( +
)Zt dt.
2
2
5) Let f (x) = x , we get
1
1
dZt = f 0 (Xt )dXt + f 00 (Xt )dhXit = 2Xt dXt + 2dhXit
2
2
2 2
2
= 2Xt (Xt dt + Xt dBt ) + Xt dt = 2Xt dt + 2Xt2 dBt + 2 Xt2 dt
= (2 + 2 )Zt dt + 2Zt dBt .

75

2.
Compute te stochastic dierential for

when

Zt = 1/Xt

where

Xt

has the

dierential

dXt = Xt dt + Xt dBt .
Proof. Let

f (x) = 1/x

and hence

f 0 (x) = 1/x2

and

f 00 (x) = 2/x3 .

By the Ito

formula we get

1
1
1 2
dZt = f 0 (Xt )dXt + f 00 (Xt )dhXit = 2 dXt +
dhXit
2
Xt
2 Xt2
1 2 2
2
Xt dt + Xt dBt
dt + dBt
+

X
+
dt
=
dt
=

t
Xt2
Xt3
Xt
Xt
= ( + 2 )Zt dt Zt dBt .

3.
Let

be a Brownian motion and

{Ft }

be the ltration generated by

B.

Show

by using stochastic calculus that the following processes are martingales.


1.

Bt2 t.

2.

eBt

2 t
2

Proof. If the Ito dierential only has an

the form

0dt + (. . .)dBt ,

dBt -part,

i.e. that dierential looks on

and the integrand is a member of the class

dened

in Denition 13.1, then we may be certain that the process is a martingale by


Proposition 13.11. 1)

1
d(Bt2 t) = 2Bt dBt + 2dt dt = 2Bt dBt
2
and since

Z t
Z t
Z t
t2
2
2
sds =
E[
E[Bs ]ds =
< ,
Bs ds] = {Fubini} =
2
0
0
0
2Bt V ,

and hence

Bt2 t

is a martingale.

2)

d(eBt

2 t
2

)=

2 t
2 Bt 2 t
1 2 Bt 2 t
2 dt + eBt 2 dB +
2 dt
e
e
t
2
2

= eBt

2 t
2

dBt .

And since

Z t
2
2
e2Bs s ds] = {Fubini} =
E[e2Bs s ]ds
0
0
0
Z t
Z t
2
2
2
e4 s e s ds < , 0 t <
E[e2Bs ]e s ds = {Bs N (0, s)} =
=
Z

eBs

E[

2 s
2

2

ds] = E[

0
hence

2
Bt 2 t

is a matingale.

76

4.
Check whether the following processes are martingales with respect to the ltration generated by
1.

Xt = Bt + 4t

2.

Xt = Bt2

3.

Xt = t2 Bt 2

4.

Xt = Bt Bt

(1)

(2)

Bt .

Rt
0

rBr dr

where

(1)

(2)

(Bt , Bt )

is a two dimensional Brownian motion.

Proof. The proofs may (at least) be done in two ways, one by checking the

martingale property of the process using a time

s < t, and the other is to do the

stochastic dierential of the process and the use the Martingale Representation
Theorem (Theorem 15.3) to conclude whether the process is a martingale or
not. We will do both. When using the Martingale Representation Theorem we
should also prove that the integrand is a member of the class

V,

which we omit

in the solutions.
1) E[Xt |Fs ] = E[Bt + 4t|Fs ] = Bs + 4t 6= Bs + 4s hence Bt + 4t is not an
Ft -martingale.
dXt = dBt + 4dt 6= g(s, )dBt hence Bt + 4t is not an Ft -martingale.
2
2
2
2) E[Xt |Fs ] = E[Bt |Fs ] = E[(Bt Bs + Bs ) |Fs ] = E[(Bt Bs ) |Fs ] +
2
2
2
2
E[Bs |Fs ] = t s + Bs 6= Bs hence Bt is not an Ft -martingale.
dXt = 2Bt dBt + dt 6= g(s, )dBt hence Bt2 is not an Ft -martingale.
3)

Z t
Z t
E[Xt |Fs ] = E[t2 Bt 2
rBr dr|Fs ] = t2 Bs 2E[
rBr dr|Fs ]
0
0
Z t
Z s
Z t
= t 2 Bs 2
rE[Br |Fs ]dr = t2 Bs 2
r E[Br |Fs ] dr 2
r E[Br |Fs ] dr
| {z }
| {z }
0
0
s
= t2 Bs 2

Z
rBr dr 2Bs

=Br
Z s

rdr = s2 Bs 2

=Bs

rBr dr = Xs
0

Rt
t2 Bt 2 0 rBr dr is an Ft -martingale.
Rt
dXt = 2tBt dt + t2 dBt 2tBt dt = t2 dBt hence t2 Bt 2 0 rBr dr

hence

is an

Ft -

martingale.

(1) (2)
(1)
(2)
4) E[Xt |Fs ] = E[Bt Bt |Fs ] = {independent} = E[Bt |Fs ]E[Bt |Fs ] =
(1) (2)
(1) (2)
Bs Bs hence Bt Bt is an Ft -martingale.
(1)
(2)
(2)
(1)
(2)
(1)
(2)
(2)
(1)
dXt = Bt dBt + Bt dBt + dhB (1) , Bt i = Bt dBt + Bt dBt hence
(1) (2)
Bt Bt is an Ft -martingale.

5.
Let

be the solution to the SDE

dXt = Xt dt + dBt , X0 = x0
where

, , x0

are constants and

is a brownian motion.

77

1. determine
2. Determine

E[Xt ].
V (Xt ).

3. Determine the solutionto trhe SDE.


Proof. 1)

Z
dBs ] = x0 + E[

Xs ds +

E[Xt ] = E[x0 +

dBs ]
Xs ds] + E[
| 0{z }
=0

Xs ds]

= x0 + E[
0

which leads to the ordinary dierential equation


d
dt

E[Xt ] = E[Xt ]

E[X0 ] = x0
E[Xt ] = x0 et
2
2
2
t
2) V (Xt ) = E[Xt ] (E[Xt ]) = E[Xt ] x0 e
so we need to nd an explicit
2
2
expression for E[Xt ]. Looking at the stochastic dierential of Xt we get

which has the solution

dXt2 = 2Xt dXt + dhXit = 2Xt (Xt dt + dBt ) + 2 dt


= (2Xt2 + 2 )dt + 2Xt dBt
from which we have

Xt2 = x20 +

(2Xt2 + 2 )dt + 2

Xt dBt .
0

E[Xt2 ]

becomes

E[Xt2 ] = E[x20 +

(2Xt2 + 2 )dt + 2

Z t
2Xt2 dt] + 2 t
Xt dBt ] = x20 + E[
0

(6)

Z t
Z t
2
+ 2 E[
Xt dBt ] = {Fubini} = x0 +
2E[Xt2 ]dt + 2 t.
0
0
|
{z
}
=0 (Mg)

Let

g(t) = E[Xt2 ],

the problem (6) leads to the ordinary dierential equation


d
dt

Consider the function


d
dt

g(t) = 2g(t) + 2 .

e2t g(t).

Dierentiating

e2t g(t)


e2t g(t) = 2g(t)e2t + e2t

78

d
dt

we get

g(t) = e2t 2 .

e2t
t

e2s ds .

Integrating both sides and multiplying both sides by


g(t) = e2t c + 2

we get

is determined by the initial condition

for

E[Xt ]

g(0) = x20

as

c = x20 .

And the formula

is

Z t


e2t 1
E[Xt ] = e2t x20 + 2
e2s ds = x20 e2t + 2
2
0
2t

V (Xt ) = E[Xt2 ] (E[Xt ])2 = 2 e 21 .


t
the dierential of e
Xt yields


d et Xt = et Xt dt + et dXt = et dBt .

so the variance is
3) Taking

Integrating both sides of (7) and multiplying both sides with

Xt = et c +

et dBt

et

(7)
gives

0
where

is determined by the initial condition

X0 = x0

to be

c = x0 .

The

solution of the SDE is therefore given by

Z t


et dBt
Xt = et x0 +
0

Xt

is the so called Ornstein Uhlenbeck process.

6.
Let

h(t)

be a deterministic function and dene the process

Z
Xt =

Xt

as

h(s)dBs .
0

Show that

Xt N (0,

Rt
0

h2 (s)ds)

by showing that

E[eiuXt ] = e

u2
2

Rt
0

h2 (s)ds

(8)

Rt

h2 (s)ds) which
0
is a unique transformation, therefore proving (8) is equal to proving that Xt
Rt
N (0, 0 h2 (s)ds).
iuXt
Let Yt = e
. Since dXt = h(t)dBt we get
Proof. Recall that (8) is the characteristic function of

N (0,

u2
u2
Yt dhXit = iuYt h(t)dBt Yt h2 (t)dt.
2
2
Rt
R
u2 t
2
Y h(s)dBs 2 0 Ys h (s)ds since Y0 = 1. The expected value
so Yt = 1 + iu
0 t
of Yt is
Z t
Z
Z t
u2 t
2
E[Yt ] = E[1 + iu
Yt h(s)dBs
Ys h (s)ds] = 1 + iu E[
Yt h(s)dBs ]
2 0
0
| 0 {z
}
dYt = iuYt dXt

=0

(9)

Z t
Z
u
u2 t
2
E[
Ys h (s)ds] = {Fubini} = 1
E[Ys ]h2 (s)ds.
2
2 0
0
2

79

(10)

Let

g(t) = E[Yt ]

and (9) may be written as the ordinary dierential equation

t
u2
g(t) = h2 (s)g(s)
dt
2
g(0) = 1
g(t) = e

which has the solution

u2
2

Rt
0

h2 (s)ds
u2
2

Rt

Rt

h2 (s)ds).

E[eiuXt ] = e
from which we conclude that

Xt N (0,

hence

h2 (s)ds

7.
Let

X, Y

satisfy the following system of SDE's

X 0 = x0

dYt = Yt dt Xt dBt ,

Y 0 = y0

Rt = Xt2 + Yt2

1. Show that
2. Compute

dXt = Xt dt + Yt dBt ,

E[Xt ], E[Yt ]

is deterministic.

and Cov(Xt , Yt ).

Proof. We rst calculate the stochastic dierentials of

Xt2

and

Yt2 .

dXt2 = 2Xt dXt + dhXit = 2Xt2 dt + 2Xt Yt dBt + Yt2 dt


dYt2 = 2Yt2 dt 2Yt Xt dBt + Xt2 dt
so we may write

Rt

as

Yt2

Xt2

Xt2 ds

+2
Xs Ys dBs +
= x0 + 2
Ys2 ds
+
0
0
0
Z t
Z t
Z t
Ys Xs dBs +
Xs2 ds = x0 + y0
+ y0 + 2
Ys2 ds 2
0
0
0
Z t
Z t
Z t
Rs ds.
Ys2 ds = (1 + 2)
+ (1 + 2)
Xt2 ds + (1 + 2)

Rt =

Let

g(t) = Rt ,

(11)

then (11) can be written as the ordinary dierential equation

d
g(t) = (1 + 2)g(s)
dt
g(0) = x0 + y0
with the solution

y0 )e

(1+2)t

g(t) = (x0 + y0 )e(1+2)t .

It has been shown that

2) rewriting

Xt

and

Yt

on integral form we get

Xt = x0 +

Xs ds +
Ys dBs
0
0
Z t
Z t
Yt = y0 +
Ys ds
Xs dBs .
0

80

Rt = (x0 +

we get

Z
Xs ds +

E[Xt ] = E[x0 +

Z t
Z t
Ys dBs ]
Xs ds] + E[
Ys dBs ] = x0 + E[
0
| 0 {z
}
=0

E[Xs ]ds

= x0 +
0

Z
Ys ds

E[Yt ] = E[y0 +
0

Z t
Z t
Xs dBs ] = y0 + E[
Ys ds] E[
Xs dBs ]
0
0
{z
}
|
=0

Z
= y0 +

E[Ys ]ds.
0

This gives two ordinary derential equations solved in the same manner as in
part 1) and gives the solutions

E[Xt ] = x0 et
E[Yt ] = y0 et .
Since the covariance is Cov(Xt , Yt )

E[Xt Yt ].

= E[Xt Yt ]E[Xt ]E[Yt ] we need to cvalculate


Xt Yt ,

For this purpose, consider the stochastic dierential of

dXt Yt = Xt dYt + Yt dXt + dhX, Y it = Yt (Xt dt + Yt dBt ) + Xt (Yt dt Xt dBt )


Xt Yt dt = (2 1)Xt Yt dt + Yt2 dBt Xt2 dBt .
Hence

Xt Yt = x0 y0 + (2 1)

Rt
0

Xs Ys ds +

Rt
0


Ys2 Xs2 dBs

and we get

Z t
Z t

E[Xt Yt ] = E[x0 y0 + (2 1)
Xs Ys ds +
Ys2 Xs2 dBs ]
0
0
Z t
Z t

2
= x0 y0 + (2 1)E[
Xs Ys ds] + E[
Ys Xs2 dBs ] = {Fubini}
0
{z
}
| 0

(12)

(13)

=0

Z
= x0 y0 + (2 1)

E[Xs Ys ]ds.

(14)

0
Let

g(t) = E[Xt Yt ], then solving (12) amounts to solving the ordinary dierential

equation

d
g(t) = (2 1)g(t)
dt
g(0) = x0 y0
which has the solution

g(t) = x0 y0 e(21)t

hence

E[Xt Yt ] = x0 y0 e(21)t

and

we get
Cov(Xt , Yt )


= E[Xt Yt ] E[Xt ]E[Yt ] = x0 y0 e(21)t x0 y0 e2t = x0 y0 e2t et 1 .

81

8.
Let

and

be processes given by the SDE's

(1)

dXt = Xt dBt
dYt = Yt dt +
where
tions.

, , , are constants
Compute E[Xt Yt ].

Proof. Start by dierentiating

(2)

+ Xt dBt ,

(1)
Yt dBt ,

and

Xt Yt

X 0 = x0

Y0 = y0

B (1) , B (2)

are independent Brownian mo-

to get

(1)

d(Xt Yt ) = Xt dYt + Yt dXt + hX, Y it = Xt (Yt dt + Yt dBt )


(1)

+ Yt (Xt dBt

(1)

(2)

+ Xt dBt ) + Xt Yt dt = ( + )Xt Yt dt + ( + )Xt Yt dBt

(2)

+ Xt Yt dBt .
Using the initial condition

Z
Xt Yt = x0 y0 + ( + )

X0 Y0 = x0 y0

we have

Z
Xs Ys ds + ( + )

0
so

E[Xt Yt ]

Xs Ys dBs(1)

Xs Ys dBs(2)

becomes

(15)
E[Xt Yt ] = E[x0 y0 + ( + )
Xs Ys ds + ( + )
Xs Ys dBs(1)
0
0
Z t
Z s
Z t
Xs Ys ds] + ( + ) E[
+
Xs Ys dBs(1) ]
Xs Ys dBs(2) ] = x0 y0 + ( + )E[
0
0
0
|
{z
}

=0

(16)

Z
+ E[
|
Let

Xs Ys dBs(2) ] = {Fubini} = x0 y0 + ( + )
{z
}

E[Xs Ys ]ds.

(17)

=0

g(t) = E[Xt Yt ],

then (15) gives the following ordinary dierential equation

d
g(t) = ( + )g(t)
dt
g(0) = x0 y0
which has the solution

g(t) = x0 y0 e(+)t

82

hence

E[Xt Yt ] = x0 y0 e(+)t .

References
General Probability:
1.
2.
3.
4.
5.
6.
7.
8.

Ross, S.M. (1980). Introduction to Probability Models, Second Edition. Academic Press, New York.
(Chapters 1, 2 and 3)
Breiman, L. (1968). Probability. Addison-Wesley, Reading, Massachusetts.
Billingsley, P. (1979). Probability and Measure. John Wiley, New York.
Feller, W. (1957). An Introduction to Probability Theory and Its Applications, Vol. 1. John Wiley &
Sons, New York.
Feller, W. (1971). An Introduction to Probability Theory and Its Applications, Vol. 2. John Wiley &
Sons, New York.
Renyi, A. (1970). Foundations of Probability. Holden-Day, San Francisco, California.
Chung, K.L. (1968). A Course in Probability. Harcourt, Brace and World, New York.
Gnedenko, B.V. and A.N. Kolmogorov (1954). Limit Distributions for Sums of Independent Random
Variables. Addison-Wesley, Reading, Massachusetts (Translation from Russian).

General Stochastic Processes:


9.
10.
11.
12.
13.
14.
15.

Cinlar, E. (1975). Introduction to Stochastic Processes. Prentice-Hall, Englewood Cliffs, New Jersey.
Cox, D.R. and H.D. Miller (1965). The Theory of Stochastic Processes. John Wiley & Sons, New York.
Doob, J.L. (1953). Stochastic Processes. John Wiley & Sons, New York.
Taylor, H.M. and S. Karlin (1984). An Introduction to Stochastic Modeling. Academic Press, New York.
Parzen, E. (1962). Stochastic Processes. Holden-Day, San Francisco, California.
Prabhu, N.U. (1965). Stochastic Processes. Macmillan, New York.
Spitzer, F. (1964). Random Walks. Van Nostrand. Princeton, New Jersey.

Discrete-time Markov Chains:


16. Ross, S.M. (1980). Introduction to Probability Models, Second Edition. Academic Press, New York.
(Chapter 4)
17. Karlin, S. and H. Taylor (1975). A First Course in Stochastic Processes, Second Edition. Academic
Press, New York. (Chapters 2 and 3)
18. Taylor, H.M. and S. Karlin (1984). An Introduction to Stochastic Modeling. Academic Press, New York.
Continuous-time Markov Chains :
19. Ross, S.M. (1980). Introduction to Probability Models, Second Edition. Academic Press, New York.
(Chapter 6)
20. Ross, S.M. (1983). Stochastic Processes. John Wiley & Sons, New York.
21. Taylor, H.M. and S. Karlin (1984). An Introduction to Stochastic Modeling. Academic Press, New York.
Stability for General State Space Markov Chains:
22. S.P. Meyn and R.L. Tweedie (1993). Markov Chains and Stochastic Stability. Springer-Verlag, New
York.
Stochastic Control
23. D. Bertsekas (1995). Dynamic Programming and Optimal Control. Volume 2. Athena, MA.
24. W. Fleming and R.W. Rishel (1975). Deterministic and Stochastic Optimal Control. Springer-Verlag.
25. W. Fleming and H.M. Soner (1992 ). Controlled Markov Processes and Viscosity Solutions. SpringerVerlag,
26. M. L. Puterman (1994). Markov Decision Processes. Wiley.
Diffusion Approximations and Brownian motion:
27. Karlin, S.and H. Taylor (1975). A Second Course in Stochastic Processes, Second Edition. Academic
Press, New York.
28. Taylor, H.M. and S. Karlin (1984). An Introduction to Stochastic Modeling. Academic Press, New York.
29. Bharucha-Reid, A.T. (1960). Elements of the Theory of Markov Processes and their Applications.
McGraw-Hill, New York

83

Renewal Theory:
30. Cox, D.R. (1962). Renewal Theory. Methuen, London.
31. Feller, W. (1971). An Introduction to Probability Theory and Its Applications, Vol. 2. John Wiley &
Sons, New York.
32. Asmussen, S. (2003). Applied Probability and Queues. 2nd ed, Springer.
33. Taylor, H.M. and S. Karlin (1984). An Introduction to Stochastic Modeling. Academic Press, New York.
Queueing Theory:
34.
35.
36.
37.
38.

Asmussen, S. (2003). Applied Probability and Queues. 2nd ed, Springer.


Gross, D. and C. Harris (1985). Fundamentals of Queueing Theory, Second Edition. Wiley, New York.
Cox, D.R. and W.L. Smith (1961). Queues.? Methuen, London.
Gross, D. and C. Harris (1985). Fundamentals of Queueing Theory, Second Edition. Wiley, New York.
Tijms, H. (1986). Stochastic Modeling and Analysis-A Computational Approach. John Wiley & Sons,
New York.

Algorithms
39. Tijms, H. (1986). Stochastic Modeling and Analysis-A Computational Approach. John Wiley & Sons,
New York.
40. Bremaud, P. Markov Chains: Gibbs Fields, Monte Carlo Simulation, and Queues. Springer.

84

You might also like